Nephrology: Specialty Review and Self-Assessment (StatPearls Review Series Book 144) [2017 Edition]

StatPearls Publishing Review Series strives to help you learn the pearls for Nephrology. This text contains 800 multiple

1,270 228 2MB

English Pages 838 Year 2016

Report DMCA / Copyright

DOWNLOAD FILE

Polecaj historie

Nephrology: Specialty Review and Self-Assessment (StatPearls Review Series Book 144) [2017 Edition]

Table of contents :
Please Contribute......Page 2
Learn More......Page 3
Introduction......Page 4
Editors in Chief......Page 9
Contributing Editors......Page 10
Authors......Page 13
Grammar Editors......Page 18
Section 10 ( Questions 361 - 400 )......Page 19
About StatPearls......Page 826

Citation preview

Become An Author or Editor StatPearls is an academic teaching project developed by thousands of health professionals worldwide. Our goal is to provide inexpensive, up to date, peer reviewed study questions for all healthcare specialties. These questions are available in Apps, eBooks, and an online learning system that includes the opportunity to earn CME/CE. We need your help improving questions. Please identify questions that need revision by flagging a question. In addition, please consider becoming an author or editor. In as little as 2-3 hours you can make a substantial contribution to medical education and you will receive free CME/CE for your contribution! Please go to www.statpearls.com to sign up.

Learn More Want to learn more? Please visit our online learning system at www.statpearls.com for additional FREE questions, color pictures, and more detailed explanations.

Introduction NOTE: The intent of StatPearls review books is to identify knowledge deficits and assist you in the learning process. Review books are not intended to be a source of the knowledge base of medicine. The authors or editors do not warrant the information is complete or accurate. The reader is encouraged to verify each answer in several references. All drug indications and dosages should be verified before administration. Copyright 2017 StatPearls Publishing, LLC. All rights reserved, including the right of reproduction, in whole or in part, in any form. The authors and editors would like to extend special thanks to Richard Miller, Tiffany Sneden, Erin Hughes, and Gerson Cordero Rubio for their editorial support and Susan Oliver for editorial team management.

http://www.StatPearls.com

Congratulations! This StatPearls book will help improve your fund of knowledge. Our books are designed to help you identify and resolve knowledge deficits. To achieve this goal the text is written in a multiple-choice format with answers and explanations. Explanations contain additional information intended to reinforce your knowledge. Emphasis has been placed on covering facts that are easily overlooked, quickly forgotten, and often show up in specialty reviews. For further information, all explanations are electronically linked to eMedicine (free), Google (free), or UpToDate (subscription). We have no affiliation with these medical websites. We suggest you note all questions answered incorrectly to allow you to rapidly review the content prior to your exam. We appreciate your comments, suggestions, and criticisms. Some answers and explanations may be in variance with your own knowledge. This is usually attributable to variability among sources; however, please make us aware of any potential errors you find as we update our books often. We appreciate your input in regard to format, content, or presentation. Please contact us at [email protected] or better yet, click on the flag button after each question to give us specific input on any errors or omissions. We look forward to hearing from you. Good luck with learning your specialty!

StatPearls eBooks StatPearls books were created exclusively for use on eBook readers. Because we don't publish our books in paper format, we have the luxury of optimizing them for eBook use. How are our books optimized for the viewing on an eBook Reader?

1. Efficient User Interface: Each multiple choice question is on a page by itself so you can see the question and four multiple choice answers, but you can't see the correct answer until you flip the page. This would be a complete waste of paper in an ordinary book, but it makes for an easy to use eBook. You don't have to cover up the correct answer with a piece of paper. Most devices have a dedicated button for page turning so with the click of a button, you can see the correct answer.

2. Research Links: After each explanation we include web links to search for more information on the key concepts for that question. If you miss a question, this is when you probably want to do more research on the question concepts. This would be onerous and require a lot of typing using a paper book, but is a single click on an eBook reader, PC, or Mac. Our customers have asked for many search options, and we have chosen the following for "assisted research": eMedicine (now Medscape/WebMD) - http://emedicine.medscape.com/ UpToDate - http://www.uptodate.com Google - http://www.google.com/ We have no affiliation with any of these three information providers. Also, to look at more than one page you may have to sign up for WebMD and UpToDate. Google requires no signup.

3. Flag a Question: Medical standards and drug standards change. Also, human errors can occur resulting in questions that have issues. StatPearls strongly believes in Continuous Quality Improvement (CQI). Flagging a question gives you, the medical professional, the ability to tell us if something is wrong with a question and we will correct it and get back to you with an updated book (providing you give us a phone number or email address.) If you don't provide us with contact information, you can check for a book update using the normal methods from the books store and you will soon find an update with the question corrected by one of our health professionals.

4. Volume of Questions: When we make a book we publish every question we have on a topic (up to 2,000 per book). This would be ridiculously expensive to do with traditional book publishing, but in the eBook world it costs no more to publish 2,000 questions than 1,000 questions. You will find that in most of our specialties, we dwarf the competition with breadth

of material. This makes our books "just what the doctor ordered" for those wishing to go the extra mile when studying. STATPEARLS AUTHORS AND EDITORS Over 3,000 physician, nursing, and allied health professionals authored a database of over a million classified multiple-choice questions and explanations. Each question has been peer reviewed by two health professionals and a pharmacist. All questions are reviewed annually and updated through a peer review process. WE APPRECIATE YOUR COMMENTS! We appreciate comments, suggestions, corrections and additional contributing authors and editors. Please e-mail us at: [email protected].

Nephrology Specialty Review and Self-Assessment StatPearls Book ID: MD_NEPHROLOGYSPECIALTY_REVIEW_AND_SELF_ASSESSMEN T-2016-10-21 Version: 10/21/2016

Editors in Chief Vijay Lapsia, MBBS, MD; Assistant Professor; Med Dir of Mount Sinai Kidney Center; Icahn School of Medicine; USA, New York, NY, USA Malvinder Parmar, MB, MS, FRCP, FASN; Professor; Medical Director, Internal Medicine, TDH; Timmins & District Hospital; Timmins, Canada Pranay Kathuria, MD; Professor; Division Chief & Fellowship Director; Uni of Oklahoma College of Medicine; USA, Tulsa, OK, USA

Contributing Editors William Alley, MD; Assistant Professor; Clerkship Director; Wake Forest School of Medicine; Winston Salem, NC, USA Syed Rizwan Bokhari, BS, MBBS, MD, FCPS; Nephrology Fellow; Consultant; Tulane Uni, Uni Med Center New Orleans; New Orleans, LA, USA Michael Bono, MD; Vice Chair, Department of Emergency Med; Eastern Virginia Medical School; Suffolk, VA, USA Steven Carsons, MD; Professor; Chief, Rheumatology; Winthrop University Hospital; Mineola, NY, USA Karen Garfield, MD, MS, BA; Body Imaging Radiologist; Mount Sinai; NY, NY, USA Alberto Goizueta, BS, MS, MD; Medical Resident; Boston Uni at Roger Williams Med Center; Providence, RI, USA Abhinav Goyal, MD; Resident; Einstein Medical Center, Philadelphia; Philadelphia, PA, USA Louis Imbriano, MD; Assistant Professor of Clinical Medicine; Winthrop University Hospital; Mineola, NY, USA Pranay Kathuria, MD; Professor; Division Chief & Fellowship Director; Uni of Oklahoma College of Medicine; Tulsa, OK, USA

Ina Kim, MD, MPH; Assistant Professor; University of Nebraska Medical Center; Omaha, NE, USA Joshua Kirkpatrick, BS; Medical Student; The University of Auckland; Auckland, New Zealand Vijay Lapsia, MBBS, MD; Assistant Professor; Med Dir of Mount Sinai Kidney Center; Icahn School of Medicine; New York, NY, USA Ashley Lessmeier, MD; PGY2; Carolinas Medical Center; Charlotte, NC, USA Abeera Mansur, MD; Assistant Professor; Kearney, NE, USA Roy Morcos, MD; Associate Professor; St. Elizabeth Boardman Hospital; Boardman, OH, USA Sadaf Munir, MBBS, DCH; Clinical Externship; Morganville, NJ, USA Wes Ogilvie, MPA, JD; Assistant Professor; Paramedic; University of Nevada; Austin, TX, USA Malvinder Parmar, MB, MS, FRCP, FASN; Professor; Medical Director, Internal Medicine, TDH; Timmins & District Hospital; Timmins, Canada Scott Plantz, MD; Associate Professor; University of Louisville; Louisville, KY, USA

Indika Ranasinghe, BS, MD; PGY3 Internal Medicine; Waikato District Health Board; Hamilton, New Zealand Helbert Rondon, MD; Assistant Professor of Medicine; University of Pittsburgh; Pittsburgh, PA, USA Juan Carlos Sánchez-Manso, MD; Universidad de Valladolid; Valladolid, Spain Jessica Stempel, MD; Medical Resident; Albert Einstein Medical Center; Philadelphia, PA, USA Stephanie Sullivan, MD; Resident; Uni of IL College of Med, OSF Med Center; Peoria, IL, USA Francisco Talavera, PharmD, PhD; Assistant Professor; University of Nebraska Medical Center; Omaha, NE, USA Imani Thornton, MD; Clinical Professor; Wayne State University Physician Group; West Bloomfield, MI, USA Matthew Warner, MD, CPE; Vice Chair Dept of EM, Clinical Faculty; Rowan Univ SOM, Inspira Health Network; Mullica Hill, NJ, USA

Authors William Alley, MD; Assistant Professor; Clerkship Director; Wake Forest School of Medicine; Winston Salem, NC, USA Subhankar Bandyopadhyay, MD, MBA; Associate Professor; Driscoll Children's Hospital, PSHMC; Hummelstown, PA, USA Steve Bhimji, BSci Pharm, MS, MD, PhD; Clinical Instructor (Retired); Toronto, Canada Syed Rizwan Bokhari, BS, MBBS, MD, FCPS; Nephrology Fellow; Consultant; Tulane Uni, Uni Med Center New Orleans; New Orleans, LA, USA Michael Bono, MD; Vice Chair, Department of Emergency Med; Eastern Virginia Medical School; Suffolk, VA, USA Zack Brady, BS, MS, MD; Resident; Western Michigan University; Portage, MI, USA Joshua Bucher, MD; Assistant Professor; Assistant EMS Medical Director; Rutgers - RWJMS; New Brunswick, NJ, USA Jean Bustamante, MD; PGY-3; Einstein Medical Center; Philadelphia, PA, USA Sylvia Cardounell; SUNY Upstate Medical University; Syracuse, NY, USA

Steven Carsons, MD; Professor; Chief, Rheumatology; Winthrop University Hospital; Mineola, NY, USA Casey Ciresi, BSN; Clinical Instructor; University of Louisville; Louisville, KY, USA Adam DeFoe, MD; Assistant Professor; University of Nebraska Medical Center; Omaha, NE, USA Chaddie Doerr, BSN, MS, CRNA; Clinical Instructor; Norton Health Systems; Treasure Island, FL, USA Scott Dulebohn, MD; Associate Professor; ETSU, Meharry, LMU, MSHA; Johnson City, TN, USA Jessie Dunne, PharmD; Assistant Professor; Clinical Pharmacy Specialist; University of Kentucky; Lexington, KY, USA Karen Garfield, MD, MS, BA; Body Imaging Radiologist; Mount Sinai; NY, NY, USA Alberto Goizueta, BS, MS, MD; Medical Resident; Boston Uni at Roger Williams Med Center; Providence, RI, USA Sherry Gossman, BSN; Clinical Instructor (Retired); Elkhorn, NE, USA William Gossman, MD; Professor; Creighton University; Elkhorn, NE, USA

Abhinav Goyal, MD; Resident; Einstein Medical Center, Philadelphia; Philadelphia, PA, USA James Hughes, MD; Assistant Professor (Retired); Evanston, IL, USA Phillip Hynes, MD; Private Practice; Lincoln, NE, USA Louis Imbriano, MD; Assistant Professor of Clinical Medicine; Winthrop University Hospital; Mineola, NY, USA Louis Imbriano, MD; Clinical Assistant Professor; Firm Chief H5; Winthrop University Hospital; Mineola, NY, USA Amer Kechli, MD; Associate Professor; Temple University; Philadelphia, PA, USA Ina Kim, MD, MPH; Assistant Professor; University of Nebraska Medical Center; Omaha, NE, USA Joshua Kirkpatrick, BS; Medical Student; The University of Auckland; Auckland, New Zealand Sidney Lane, MS, MD, PhD; Professor Emeritus; Retired; Espoo, Finland Stephen Leslie, MD; Associate Professor of Surgery, Urology; Creighton University Medical Center; Omaha, NE, USA Whitney Lynch, BS, DO; Internal Medicine Resident; Merit Health Wesley; Hattiesburg, MS, USA

Kenneth Metcalf, MD, PhD; Professor Emeritus; Dept of Anatomy, Embryology, & Genetics; UNMC; Omaha, NE, USA Roy Morcos, MD; Associate Professor; St. Elizabeth Boardman Hospital; Boardman, OH, USA Sadaf Munir, MBBS, DCH; Clinical Externship; Morganville, NJ, USA Asif Noor, MD; Pediatric Infectious Diseases Physician; Winthrop University Hospital; Mineola, NY, USA Wes Ogilvie, MPA, JD; Assistant Professor; Paramedic; University of Nevada; Austin, TX, USA Malvinder Parmar, MB, MS, FRCP, FASN; Professor; Medical Director, Internal Medicine, TDH; Timmins & District Hospital; Timmins, Canada Scott Plantz, MD; Associate Professor; University of Louisville; Louisville, KY, USA Nandini Devi Ramaswamy, MBBS, MD; PGY 1; Roger Williams Medical Center; Providence, RI, USA Indika Ranasinghe, BS, MD; PGY3 Internal Medicine; Waikato District Health Board; Hamilton, New Zealand Helbert Rondon, MD; Assistant Professor of Medicine; University of Pittsburgh; Pittsburgh, PA, USA

Ayan Sabih, MBBS; Medical Registrar; Waikato DHB; Hamilton, New Zealand Nathan Samras, MD, MPH; Assistant Professor; Savannah Memorial Hosp, Mercer Med Sch; Savannah, GA, USA Frank Smeeks, MS, MD, MBA; Private Practice; Hickory, NC, USA Jessica Stempel, MD; Medical Resident; Albert Einstein Medical Center; Philadelphia, PA, USA Stephanie Sullivan, MD; Resident; Uni of IL College of Med, OSF Med Center; Peoria, IL, USA Francisco Talavera, PharmD, PhD; Assistant Professor; University of Nebraska Medical Center; Omaha, NE, USA Pranav Virmani, MD; Partner & Owner of Commonwealth Emer Med; Inova Loudoun Hospital; Leesburg, VA, USA Matthew Warner, MD, CPE; Vice Chair Dept of EM, Clinical Faculty; Rowan Univ SOM, Inspira Health Network; Mullica Hill, NJ, USA Scott Westphal, MD; Assistant Professor; University of Nebraska Medical Center; Omaha, NE, USA Micah Wittler, DO, MS; Associate Professor; Western University of Health Sci; Redlands, CA, USA

Grammar and Illustration Editors Susan Oliver, BA Tiffany Sneden, BA Richard Miller Erin Hughes Gerson Cordero Rubio

Table of Contents Please Contribute Learn More Introduction Editors in Chief Contributing Editors Authors Section 1 ( Questions 1 - 40 ) Section 2 ( Questions 41 - 80 ) Section 3 ( Questions 81 - 120 ) Section 4 ( Questions 121 - 160 ) Section 5 ( Questions 161 - 200 ) Section 6 ( Questions 201 - 240 ) Section 7 ( Questions 241 - 280 ) Section 8 ( Questions 281 - 320 ) Section 9 ( Questions 321 - 360 ) Section 10 ( Questions 361 - 400 ) About StatPearls

Section 1 Question 1: How is tadalafil different from similar drugs used for erectile dysfunction?

Choices: 1. It has fewer side effects 2. It has longer duration of action 3. It is highly potent 4. It is efficacious

Answer: 2 - It has longer duration of action Explanations: The elimination half-life of tadalafil is about four-fold longer than other similar drugs. This makes it last longer clinically. It may also affect PDE11 and cause some large muscle pain in the buttocks or back which is not seen with other erectile dysfunction drugs. Half-life has no effect on either the potency nor the efficacy of a drug. It is also approved for daily use and for benign prostatic hyperplasia. Go to the next page if you knew the correct answer, or click the link images below to further research the concepts in this question (if desired).

Research Concepts: Tadalafil:

We update eBooks quarterly and Apps daily based on user feedback. Please tap flag to report any questions that need improvement.

Question 2: In a patient who is on long-term hemodialysis, the osteomalacia may be due to which of the following?

Choices: 1. Aluminum 2. Iron 3. Hypocalcemia 4. Urea

Answer: 1 - Aluminum Explanations: In patients who are on long-term hemodialysis, osteomalacia does develop and aluminum is often found to accumulate in bone. However, the majority of these studies are from animals. Patients on dialysis may complain of proximal muscle weakness, bone pain, non-healing fractures, and altered mental status. On x-ray, looser zones may be seen. Histological stains for aluminum are not always positive. If aluminum excess is confirmed, some patients may be treated with deferoxamine, which is a chelator of iron. The chelator also has an affinity for iron, thus iron supplements are needed during deferoxamine therapy. Go to the next page if you knew the correct answer, or click the link images below to further research the concepts in this question (if desired).

Research Concepts: Osteomalacia, Renal Osteodystrophy:

We update eBooks quarterly and Apps daily based on user feedback. Please tap flag to report any questions that need improvement.

Question 3: A 32-year-old African American female with sarcoidosis complains of polyuria which is found to be secondary to the sarcoidosis. Which of the following is most likely true?

Choices: 1. Vasopressin injection in a water deprivation test raises urine osmolality greater than 9% 2. There is low serum osmolality with high urine osmolality 3. Vasopressin has no affect on urine osmolality during a water deprivation test 4. There is a high serum glucose with a high urine osmolality

Answer: 1 - Vasopressin injection in a water deprivation test raises urine osmolality greater than 9%

Explanations: Sarcoidosis can cause central diabetes insipidus (DI). If vasopressin has a large affect, DI is central in origin. If vasopressin does not have an affect, DI is nephrogenic in origin. High glucose with high urine osmolality is seen in diabetes mellitus. Go to the next page if you knew the correct answer, or click the link images below to further research the concepts in this question (if desired).

Research Concepts: Diabetes Insipidus:

We update eBooks quarterly and Apps daily based on user feedback. Please tap flag to report any questions that need improvement.

Question 4: Ammonia formation in the urine is due to the action of: Choices: 1. Artinase 2. Glutaminase 3. Phosphatase 4. Histamine decarboxylase

Answer: 2 - Glutaminase Explanations: Glutamine formed in tissues from ammonia and glutamic acid is hydrolyzed in the kidney by glutaminase releasing the ammonia directly into the urine. Go to the next page if you knew the correct answer, or click the link images below to further research the concepts in this question (if desired).

Research Concepts: Ammonia:

We update eBooks quarterly and Apps daily based on user feedback. Please tap flag to report any questions that need improvement.

Question 5: In a horseshoe kidney, the fusion usually occurs at which location?

Choices: 1. The renal pelves 2. The lower poles of both kidneys 3. The upper poles of both kidneys 4. The ureteropelvic junctions

Answer: 2 - The lower poles of both kidneys Explanations: Horseshoe kidney is the most common renal fusion anomaly, seen in 1 in 400 - 800 births. Fusion occurs embryologically and is more common in males than females. Fusion is most commonly at the lower poles, where an isthmus is formed, either fibrous or parenchymal. Horseshoe kidney results in an increased risk of renal calculus formation, kidney infections and partial obstruction. This is all related to the partial stasis of urine that occurs given the mechanical positioning of the renal pelves after lower pole fusion resulting in their anterolateral displacement and rotation. Go to the next page if you knew the correct answer, or click the link images below to further research the concepts in this question (if desired).

Research Concepts: Horseshoe Kidney:

We update eBooks quarterly and Apps daily based on user feedback. Please tap flag to report any questions that need improvement.

Question 6: An asymptomatic 80 year old female has a urine culture showing greater than 100,000 E. coli. What should be done?

Choices: 1. Treat the patient and then repeat the culture 2. Treat the patient but do no further evaluation 3. Repeat the culture 4. Do nothing

Answer: 4 - Do nothing Explanations: The patient has asymptomatic bacteruria. Often, patients will have no WBCs in the urine, and the infection is transient. Symptoms of infection can include dysuria, fever, abdominal pain, urinary frequency, fatigue, mental status changes, or urinary urgency. No treatment is needed for asymptomatic bacteruria in females. Go to the next page if you knew the correct answer, or click the link images below to further research the concepts in this question (if desired).

Research Concepts: Asymptomatic Bacteriuria:

We update eBooks quarterly and Apps daily based on user feedback. Please tap flag to report any questions that need improvement.

Question 7: A 38 year old female presents with renal colic. CT shows papillary calcification, hyperdense papilla, and ectasia of precaliceal tubules. Select the most likely diagnosis.

Choices: 1. Adult polycystic kidney disease 2. Medullary sponge kidney 3. Hyperparathyroidism causing nephrolithiasis 4. Renal dysplasia

Answer: 2 - Medullary sponge kidney Explanations: Patients with medullary sponge kidneys often make calcium stones. Presentation is most often around age 20 to 40. Progression to renal failure is rare. Pathologic analysis shows multiple very small cuboidal or columnar lined cysts. Go to the next page if you knew the correct answer, or click the link images below to further research the concepts in this question (if desired).

Research Concepts: Medullary Sponge Kidney:

We update eBooks quarterly and Apps daily based on user feedback. Please tap flag to report any questions that need improvement.

Question 8: Findings of a hexagonal crystal in the urine suggests what type of renal calculi?

Choices: 1. Cystine 2. Uric acid 3. Calcium 4. Potassium

Answer: 1 - Cystine Explanations: Cystine stones are usually hexagonal in their shape. They are responsible for less than one percent of renal stones. Go to the next page if you knew the correct answer, or click the link images below to further research the concepts in this question (if desired).

Research Concepts: Nephrolithiasis:

We update eBooks quarterly and Apps daily based on user feedback. Please tap flag to report any questions that need improvement.

Question 9: A patient with nausea and confusion following hemodialysis may have developed which of the following?

Choices: 1. Transient ischemic attack 2. Air embolus 3. Hypertension 4. Disequilibrium syndrome

Answer: 4 - Disequilibrium syndrome Explanations: Dialysis disequilibrium syndrome is caused by sudden changes in osmolality causing movement of water into the brain. Symptoms include confusion, headache, nausea, and seizure. Go to the next page if you knew the correct answer, or click the link images below to further research the concepts in this question (if desired).

Research Concepts: Dialysis, Hemodialysis:

We update eBooks quarterly and Apps daily based on user feedback. Please tap flag to report any questions that need improvement.

Question 10: What percent of patients with gout develop nephrolithiasis?

Choices: 1. 10 percent 2. 20 percent 3. 30 percent 4. 40 percent

Answer: 2 - 20 percent Explanations: Uric acid elevation is the cause of gout. Gout is characterized by swollen, painful joints. About 20 percent of these patients develop nephrolithiasis. Go to the next page if you knew the correct answer, or click the link images below to further research the concepts in this question (if desired).

Research Concepts: Nephrolithiasis:

We update eBooks quarterly and Apps daily based on user feedback. Please tap flag to report any questions that need improvement.

Question 11: Which antituberculosis/anti-BCG drug causes orange urine?

Choices: 1. Rifampin 2. Ethambutol 3. INH (Isoniazide) 4. Streptomycin

Answer: 1 - Rifampin Explanations: INH can cause skin eruptions, peripheral neuritis, and hepatitis. Ethambutol can cause renal damage and drug fever. Rifampin can cause orange urine, tears and sweat. Streptomycin can cause vestibular ototoxicity. Go to the next page if you knew the correct answer, or click the link images below to further research the concepts in this question (if desired).

Research Concepts: Rifampin:

We update eBooks quarterly and Apps daily based on user feedback. Please tap flag to report any questions that need improvement.

Question 12: A 60 year old man presents with macroscopic hematuria. CT scan with IV contrast shows a 3 cm complex cystic mass with multiple thin enhancing septations. Follow up CT scan with IV contrast 1 year later shows a 3.8 cm lesion with progressive septal thickening and nodular enhancement. Which of the following is correct?

Choices: 1. It is consistent with a Bosniak III classification and given interval growth and progressive septal thickening. Recommend surgical removal. 2. It is consistent with a Bosniak IIF classification. Recommend follow up dedicated kidney CT scan before and after IV contrast in 1 year 3. It is consistent with a Bosniak I classification and recommend no further follow up or imaging 4. It is consistent with benign multilocular cystic nephroma. Recommend no further imaging or follow up

Answer: 1 - It is consistent with a Bosniak III classification and given interval growth and progressive septal thickening. Recommend surgical removal.

Explanations: Bosniak III cystic renal masses are indeterminate and require surgical intervention. Bosniak III are thick nodular septation with measurable enhancement. Chance for malignancy for Bosniak III is 50 percent. The imaging characteristics of the initial 3.0 cm renal mass is consistent with Bosniak IIF.which are multiple thin septations with minimal enhancement. Chance of malignancy for Bosniak IIF is 5-10 percent. Bosniak I classification is a simple cyst and has a no chance of malignancy and needs no further follow up. Multilocular cystic nephroma is a multicystic benign renal lesions with thin septations that has a bimodal age distribution in 2 year old males and females in their 40's. This patient does not fit the bimodal distribution. Even if a multilocular cystic nephroma is suspected, the treatment is surgical removal give the overlap of imaging with cystic renal cell cancer. Go to the next page if you knew the correct answer, or click the link images below to further research the concepts in this question (if desired).

Research Concepts: Cancer, Renal Cell:

We update eBooks quarterly and Apps daily based on user feedback. Please tap flag to report any questions that need improvement.

Question 13: Which of the following medications is known to cause kidney stones?

Choices: 1. Raltegravir 2. Zidovudine 3. Indinavir 4. Tenofovir

Answer: 3 - Indinavir Explanations: Indinavir is a protease inhibitor similar to the actions of atazanavir. It commonly causes kidney stones. Tenofovir and zidovudine are NRTIs. Raltegravir is an integrase inhibitor. Go to the next page if you knew the correct answer, or click the link images below to further research the concepts in this question (if desired).

Research Concepts: Indinavir:

We update eBooks quarterly and Apps daily based on user feedback. Please tap flag to report any questions that need improvement.

Question 14: What is the most common urological side effect of lithium?

Choices: 1. Nephrogenic diabetes insipidus 2. Erectile dysfunction 3. Premature ejaculation 4. Urinary incontinence

Answer: 1 - Nephrogenic diabetes insipidus Explanations: Lithium can act as a potent diuretic. Prolonged use has been associated with nephrogenic diabetes insipidus which may become irreversible. End-stage renal disease (ESRD) due to chronic, long term lithium use may occur in a very small percentage of patients. Chronic lithium ingestion can also lead to resistance to ADH (antidiuretic hormone). This can cause significant polyuria in up to 40 percent of patients. Any unexplained acute nocturia in patients who use lithium chronically should be suspected of diabetes insipidus. A simple voiding diary can be helpful in detecting this. Go to the next page if you knew the correct answer, or click the link images below to further research the concepts in this question (if desired).

Research Concepts: Lithium:

We update eBooks quarterly and Apps daily based on user feedback. Please tap flag to report any questions that need improvement.

Question 15: Which of the following is NOT a disorder of the basement membrane?

Choices: 1. Alport syndrome 2. Goodpasture syndrome 3. Epidermolysis bullosa 4. Diabetic nephropathy

Answer: 4 - Diabetic nephropathy Explanations: A few diseases are due to defects in the basement membrane. Genetic defects in collagen fibers of the basement membrane can cause Alport syndrome. Goodpasture is due to deposition of antigen in the basement membrane. Epidermolysis bullosa is due to poor function of basement membrane. Go to the next page if you knew the correct answer, or click the link images below to further research the concepts in this question (if desired).

Research Concepts: Basement Membrane Disease:

We update eBooks quarterly and Apps daily based on user feedback. Please tap flag to report any questions that need improvement.

Question 16: What diuretic agent is known to cause hyperkalemia in a patient already receiving potassium supplements?

Choices: 1. Triamterene 2. Acetazolamide 3. Furosemide 4. Bumetanide

Answer: 1 - Triamterene Explanations: Triamterene is a potassium-sparing diuretic used in the treatment of congestive heart failure or high blood pressure. Since triamterene may increase potassium levels, avoid consuming large amounts of bananas, prunes, apricots, peaches, tomato juice, orange juice, and salt-substitutes as these contain high amount of potassium. Triamterene is often combined with diuretics that deplete potassium. Go to the next page if you knew the correct answer, or click the link images below to further research the concepts in this question (if desired).

Research Concepts: Triamterene:

We update eBooks quarterly and Apps daily based on user feedback. Please tap flag to report any questions that need improvement.

Question 17: Which is most likely to be observed in a patient with renal failure?

Choices: 1. Hypercalcemia 2. Hypocalcermia 3. Hyperkalemia 4. Hypokalemia

Answer: 3 - Hyperkalemia Explanations: Hyperkalemia is a common electrolyte abnormality in renal failure. Go to the next page if you knew the correct answer, or click the link images below to further research the concepts in this question (if desired).

Research Concepts: Renal Failure:

We update eBooks quarterly and Apps daily based on user feedback. Please tap flag to report any questions that need improvement.

Question 18: A 17 year-old college student comes to ER with severe diarrhea for 2 days. He tried to keep himself hydrated drinking a sports drink at home. He was in Haiti until 3 days ago where he was involved in reconstruction efforts after the 2010 earthquake. On exam: supine: BP 100/60, HR=94, standing: BP 70/40, HR=130. Jugular veins are flat. Lungs are clear to auscultation. No peripheral edema. Laboratory data: Na=129 mEq/L, K=3.1 mEq/L, TCO2=18 mmol/L, UOsm=413 mOsm/kg, UNa=25 mEq/L, UK=13 mEq/L. What is the most likely cause of hyponatremia in this patient?

Choices: 1. Psychogenic polydipsia 2. Decrease effective arterial blood volume 3. SIADH 4. Low solute intake

Answer: 2 - Decrease effective arterial blood volume Explanations: Hypovolemia is a very common cause of hyponatremia. Hypovolemia can cause a significant reduction of the effective arterial blood volume with activation of vasopressin. Hyponatremia only occurs in hypovolemia when patient drinks water or hypotonic fluids. Treatment of hypovolemic hyponatremia involves volume expansion with isotonic fluids. Go to the next page if you knew the correct answer, or click the link images below to further research the concepts in this question (if desired).

Research Concepts: Hyponatremia:

We update eBooks quarterly and Apps daily based on user feedback. Please tap flag to report any questions that need improvement.

Question 19: A young otherwise healthy female on no medications presents complaining of swelling and is found to have a BP of 150/93, periorbital edema, 3+ peripheral edema, normal cardiovascular exam and the following lab values: Creatinine 0.7 mg/dL, albumin 2.2 g/L, Urinalysis 3+ protein, 0 RBC/HPF, 0 WBC/HPF, and oval fat bodies. What should be the first diagnostic test ordered?

Choices: 1. Renal ultrasound 2. Measurement of urine protein to creatinine ratio 3. Serum protein electrophoresis 4. Test for antineutrophil cytoplasmic antibodies (ANCA)

Answer: 2 - Measurement of urine protein to creatinine ratio Explanations: This is a case of nephrotic syndrome as indicated by low serum albumin resulting in hypertension and edema. Nephrotic range proteinuria is 3+ to 4+ protein by dipstick urinalysis or a urine albumin: creatinine ratio over 2. or a total of >3 g/day. Multiple myeloma can be diagnosed by identification of Bence Jones proteins by serum and urine protein electrophoresis and it causes proteinuria that is not detected by the urine dipstick that is albumin specific. Autoimmune diseases such as systemic lupus can cause nephrotic syndrome but are typically accompanied by other systemic signs and symptoms. Go to the next page if you knew the correct answer, or click the link images below to further research the concepts in this question (if desired).

Research Concepts: Nephrotic Syndrome:

We update eBooks quarterly and Apps daily based on user feedback. Please tap flag to report any questions that need improvement.

Question 20: A patient has uric acid renal stones. Which of the following is the best prophylactic treatment for his condition?

Choices: 1. HCTZ 2. Penicillin 3. Allopurinol 4. Indomethacin

Answer: 3 - Allopurinol Explanations: Allopurinol reduces serum uric acid levels by blocking xanthine oxidase. Xanthine is far more soluble than uric acid and rarely causes any problems. Optimal serum levels of uric acid are 6 or less; optimal urinary excretion is 600 mg/day or less. Prophylactic therapy would include potassium citrate to alkalinize the urine. Go to the next page if you knew the correct answer, or click the link images below to further research the concepts in this question (if desired).

Research Concepts: Uric Acid Stones:

We update eBooks quarterly and Apps daily based on user feedback. Please tap flag to report any questions that need improvement.

Question 21: In a patient with class 1 lupus nephritis, what will biopsy of the glomeruli reveal?

Choices: 1. Mesangial immune deposits 2. Subendothelial deposits 3. Diffuse global proliferative nephritis 4. Segmental proliferative nephritis

Answer: 1 - Mesangial immune deposits Explanations: In class 1 lupus nephritis, one will only see mesangial immune deposits. As the nephritis worsens one may see subepithelial and subendothelial deposits. Segmental and diffuse proliferative lupus are seen in classes 4 and 5. Go to the next page if you knew the correct answer, or click the link images below to further research the concepts in this question (if desired).

Research Concepts: Lupus Nephritis:

We update eBooks quarterly and Apps daily based on user feedback. Please tap flag to report any questions that need improvement.

Question 22: A patient with chronic hyponatremia with a serum Na+ of 110 mEq/L is given 2 L of 0.9 NS at 100 mL/h. Which of the following sets of urine electrolytes will predict a higher rise in serum Na+?

Choices: 1. UNa=160 mEq/L and UK=40 mEq/L 2. UNa=100 mEq/L and UK=54 mEq/L 3. UNa=70 mEq/L and UK=30 mEq/L 4. UNa=40 mEq/L and UK=20 mEq/L

Answer: 4 - UNa=40 mEq/L and UK=20 mEq/L Explanations: The urine-plasma electrolyte ratio can predict serum sodium concentration response to fluid restriction and normal saline. Urine-plasma electrolyte ratio is calculated by (UNa + UK)/PNa. A low urine-plasma electrolyte ratio predicts an increase in serum sodium with fluid restriction. A urine-plasma electrolyte ratio of less than 154 mEq/L predicts an increase in serum sodium concentration with normal saline. Go to the next page if you knew the correct answer, or click the link images below to further research the concepts in this question (if desired).

Research Concepts: Hyponatremia:

We update eBooks quarterly and Apps daily based on user feedback. Please tap flag to report any questions that need improvement.

Question 23: The End-Stage Renal Disease (ESRD) Program is funded by:

Choices: 1. Local hospitals 2. Medicaid 3. Medicare 4. None of the above

Answer: 3 - Medicare Explanations: The ESRD Program is funded by Medicare. The majority (~93%) of individuals with ESRD are eligible for the program. Go to the next page if you knew the correct answer, or click the link images below to further research the concepts in this question (if desired).

Research Concepts: End Stage Renal Disease:

We update eBooks quarterly and Apps daily based on user feedback. Please tap flag to report any questions that need improvement.

Question 24: What is the most frequent neurologic complication in patients with chronic renal failure?

Choices: 1. Seizures 2. Peripheral neuropathy 3. Parkinsonism 4. Dementia

Answer: 2 - Peripheral neuropathy Explanations: Patients with chronic renal failure can develop peripheral neuropathy that is most often distal, symmetric, and mixed sensorimotor. It affects males more than females. There is often axon degeneration. Dialysis may improve the condition. Go to the next page if you knew the correct answer, or click the link images below to further research the concepts in this question (if desired).

Research Concepts: Renal Failure, Chronic:

We update eBooks quarterly and Apps daily based on user feedback. Please tap flag to report any questions that need improvement.

Question 25: Select the antiseizure medication that is dialyzable. Choices: 1. Valproic acid 2. Carbamazepine 3. Phenytoin 4. Topiramate

Answer: 4 - Topiramate Explanations: Valproic acid, carbamazepine, and phenytoin are not dialyzable. Go to the next page if you knew the correct answer, or click the link images below to further research the concepts in this question (if desired).

Research Concepts: Topiramate:

We update eBooks quarterly and Apps daily based on user feedback. Please tap flag to report any questions that need improvement.

Question 26: A patient has both lower extremities crushed after being hit by a car. On admission, the patient is restless and is complaining of pain in both legs. His blood pressure is 130/80 mm Hg and his heart rate is 90 beats per minute. Abdominal and chest x-rays are normal. Both thighs are swollen and both femurs are obviously fractured. Urinalysis reveals massive hemoglobinuria. What is the next appropriate step in his management?

Choices: 1. Ultrasound of kidneys 2. Blood transfusion 3. Normal saline with sodium bicarbonate 4. Furosemide

Answer: 3 - Normal saline with sodium bicarbonate Explanations: Vigorous hydration with isotonic crystalloid is the cornerstone of rhabdomyolysis treatment. Because injured myocytes tend to sequester a lot of fluid, fluids should be administered liberally. Urinary alkalinization to prevent acute renal failure is supported by literature. This may be combined with mannitol once the patient has been resuscitated. Loop diuretics should only be used once the patient has been hydrated. Hyperkalemia does occur and may need to be treated with glucose and insulin. Go to the next page if you knew the correct answer, or click the link images below to further research the concepts in this question (if desired).

Research Concepts: Rhabdomyolysis:

We update eBooks quarterly and Apps daily based on user feedback. Please tap flag to report any questions that need improvement.

Question 27: Which of the following statements about amphotericin is FALSE?

Choices: 1. It is toxic to the kidneys 2. It inhibits ergosterol 3. It can cause fever, chills, and vomiting 4. It is excreted by the liver

Answer: 4 - It is excreted by the liver Explanations: Amphotericin is nephrotoxic. It is excreted by the kidneys. It is usually administered over a period of 4-6 weeks and may cause pruritus, fever, and vomiting. Pretreatment with acetaminophen, meperidine, diphenhydramine, or hydrocortisone may decrease these. Other adverse effects include anorexia, chills, diarrhea, headache, hypokalemia, hypomagnesemia, hypotension, malaise, and tachypnea. Tolerability may be improved by using plain cholesteryl sulfate complex, lipid complex, or a liposomal formulation. Go to the next page if you knew the correct answer, or click the link images below to further research the concepts in this question (if desired).

Research Concepts: Amphotericin B:

We update eBooks quarterly and Apps daily based on user feedback. Please tap flag to report any questions that need improvement.

Question 28: In what condition are subepithelial humps present? Choices: 1. IgA nephropathy 2. Diabetic nephropathy 3. Berger disease 4. Post streptococcal glomerulonephritis

Answer: 4 - Post streptococcal glomerulonephritis Explanations: The histological features of post streptococcal glomerulonephritis depends on the severity of the illness. Electron microscopy often shows electron dense humps in the subepithelial space. During recovery, these subepithelial humps disappear. Granular deposits of IgG and C3 along the capillary wall are common. Go to the next page if you knew the correct answer, or click the link images below to further research the concepts in this question (if desired).

Research Concepts: Poststreptococcal Glomerulonephritis, Acute:

We update eBooks quarterly and Apps daily based on user feedback. Please tap flag to report any questions that need improvement.

Question 29: Which patients are most likely to develop HIVassociated nephropathy?

Choices: 1. Recent seroconversion 2. Long duration of disease 3. Perinatally-acquired disease 4. African Americans

Answer: 4 - African Americans Explanations: HIV-associated nephropathy is found most commonly in HIV patients with African heritage and who have low CD4 counts. Intravenous drug use is the most common method of exposure among these patients. Age and duration of illness are not risk factors. Renal disease is a relatively common complication. Go to the next page if you knew the correct answer, or click the link images below to further research the concepts in this question (if desired).

Research Concepts: HIV, Nephropathy:

We update eBooks quarterly and Apps daily based on user feedback. Please tap flag to report any questions that need improvement.

Question 30: Which of the following is true about extracorporeal shock wave therapy for renal stones?

Choices: 1. It requires stones to be present in the renal pelvis 2. It rarely requires repeated treatment regardless of stone size 3. It is more effective for stones < 2 cm in diameter 4. It requires less energy for calcium oxalate and cystine stones

Answer: 3 - It is more effective for stones < 2 cm in diameter Explanations: Lithotripsy has been used to fragment and remove kidney stones. The procedure involves placing the patient on a lithotripsy gantry so the calculus overlies a circular window in the table. The window contains the water bath and is focused on the calculus. The procedure is more effective for stones < 2 cm in diameter. Calcium oxalate and cystine stones are usually dense and require increased energy. Large stones may require repeated treatments. Previously, extracorporeal shock wave therapy was reserved primarily for renal calculi, but now it may be used for calculi within the kidney, ureter, or bladder. The therapy has a 90% chance of rendering the patient stone-free within 3 months. Go to the next page if you knew the correct answer, or click the link images below to further research the concepts in this question (if desired).

Research Concepts: Lithotripsy, Extracorporeal:

We update eBooks quarterly and Apps daily based on user feedback. Please tap flag to report any questions that need improvement.

Question 31: Which of the following medications is indicated for use in salicylate poisoning?

Choices: 1. Amphetamines 2. Sodium bicarbonate 3. Probenecid 4. Charcoal

Answer: 2 - Sodium bicarbonate Explanations: Alkalinizing the urine can enhance renal clearance of acidic drugs. Sodium bicarbonate is often used to enhance elimination of acidic drugs. Amphetamines are not used for the treatment of salicylate poisoning. Go to the next page if you knew the correct answer, or click the link images below to further research the concepts in this question (if desired).

Research Concepts: Toxicity, Salicylate (Aspirin):

We update eBooks quarterly and Apps daily based on user feedback. Please tap flag to report any questions that need improvement.

Question 32: A 55 year old is found to have marked eosinophilia with renal failure. He may have had what type of pathology?

Choices: 1. Allergic 2. Autoimmune 3. Bacterial 4. Viral

Answer: 1 - Allergic Explanations: Eosinophilia with renal failure is highly suggestive of allergic interstitial nephritis. Possibly drug induced. Go to the next page if you knew the correct answer, or click the link images below to further research the concepts in this question (if desired).

Research Concepts: Nephritis, Interstitial, Allergic:

We update eBooks quarterly and Apps daily based on user feedback. Please tap flag to report any questions that need improvement.

Question 33: 67 year-old homeless man with a PMH of alcohol dependence was admitted for right leg cellulitis. He drinks a 6-pack of beer every day and barely eats solid food. On exam: BP=120/60, HR=73, RR=12. He appears very malnourished. No JVD. Lungs are clear to auscultation. Right leg is erythematous, edematous, and tender to palpation. No edema on left leg. Neurologic exam is completely normal. Laboratory data: Na=108 mEq/L, UOsm=65 mOsm/kg. What is the most appropriate therapeutic intervention at this point?

Choices: 1. NaCl 3% IV bolus 2. NaCl 3% slow infusion 3. Tolvaptan 4. Fluid restriction

Answer: 4 - Fluid restriction Explanations: NaCl 3% should be used for patients with symptomatic hyponatremia. Low solute hyponatremia requires fluid restriction and a regular diet. Tolvaptan is indicated in hyponatremia associated with high ADH activity. Go to the next page if you knew the correct answer, or click the link images below to further research the concepts in this question (if desired).

Research Concepts: Hyponatremia:

We update eBooks quarterly and Apps daily based on user feedback. Please tap flag to report any questions that need improvement.

Question 34: What sodium level is associated with development of seizures?

Choices: 1. 115 2. 125 3. 130 4. 135

Answer: 1 - 115 Explanations: Serum sodium concentration less than 120 mEq per liter can result in seizures. Go to the next page if you knew the correct answer, or click the link images below to further research the concepts in this question (if desired).

Research Concepts: Hyponatremia:

We update eBooks quarterly and Apps daily based on user feedback. Please tap flag to report any questions that need improvement.

Question 35: Which is CORRECT regarding a simple renal cyst? Choices: 1. It is classified as Bosniak 1 classification 2. It is classified as Bosniak 2 classification 3. It can have a thin septation on CT scan 4. It has a 5% chance of being malignant.

Answer: 1 - It is classified as Bosniak 1 classification Explanations: Bosniak 1 classification is a simple renal cyst. The Bosniak classification system of renal cystic masses divides renal cystic masses into categories 1 to 4 based on imaging characteristics on contrastenhanced CT. It is useful in predicting a risk of malignancy and determining follow up or treatment. Bosniak 2 classification is a minimally complex renal cyst with thin 1mm septations. Bosniak 2 requires NO follow up imaging. A simple cyst has NO septations. A simple cyst has 0% chance of being malignant. Go to the next page if you knew the correct answer, or click the link images below to further research the concepts in this question (if desired).

Research Concepts: Cyst, Renal:

We update eBooks quarterly and Apps daily based on user feedback. Please tap flag to report any questions that need improvement.

Question 36: Which of the following is included in the recommended workup for horseshoe kidney?

Choices: 1. CT urogram (CT scan of the abdomen and pelvis without and with IV contrast) 2. Voiding cystourethrogram (VCUG) 3. Diuresis nuclear renal scan 4. All of the above

Answer: 4 - All of the above Explanations: All of the imaging studies are reasonable in a newly discovered horseshoe kidney. The CT scan gives the anatomy, the VCUG indicates reflux, and the nuclear renal scan can assess renal function and help identify UPJ obstructions. 24 hour urine testing for stone prevention is recommended in horseshoe kidney patients with a history of nephrolithiasis. Urinalysis and cultures should be checked and infections treated if necessary. Go to the next page if you knew the correct answer, or click the link images below to further research the concepts in this question (if desired).

Research Concepts: Horseshoe Kidney:

We update eBooks quarterly and Apps daily based on user feedback. Please tap flag to report any questions that need improvement.

Question 37: An 17-year-old otherwise healthy female on no medications presents complaining of swelling and is found to have a BP of 150/93, periorbital edema, 3+ peripheral edema, normal cardiovascular exam and the following lab values: Creatinine 0.7 mg/dL, albumin 2.2 g/L, Urinalysis 3+ protein, 0 RBC/HPF, 0 WBC/HPF, 1+oval fat bodies. What should be the first diagnostic test ordered?

Choices: 1. Renal ultrasound 2. Measurement of urine albumin 3. Serum protein electrophoresis 4. Test for anti-neutrophil cytoplasmic antibodies (ANCA)

Answer: 2 - Measurement of urine albumin Explanations: This is a case of nephrotic syndrome as indicated by low serum albumin resulting in hypertension and edema. Nephrotic range proteinuria is 3+ to 4+ protein by dipstick urinalysis or a urine albumin: creatinine ratio over 3.5 mg/g and the nephrotic syndrome causes oval fat bodies to be visible on microscopic analysis of urine specimens. Autoimmune diseases such as systemic lupus can cause nephrotic syndrome but are typically accompanied by other systemic signs and symptoms. Go to the next page if you knew the correct answer, or click the link images below to further research the concepts in this question (if desired).

Research Concepts: Nephrotic Syndrome:

We update eBooks quarterly and Apps daily based on user feedback. Please tap flag to report any questions that need improvement.

Question 38: Which of the following is seen in 25% of patients with polycystic kidney disease?

Choices: 1. Floppy heart valve 2. Normal ultrasound of the kidneys after the age of 30 3. Kidney failure by age 20 4. Aneurysm

Answer: 1 - Floppy heart valve Explanations: About 25% of patients with polycystic kidney disease will have a floppy valve in the heart. It is rare for someone with a normal ultrasound of the kidneys after the age of 30 to manifest polycystic kidney disease. About 50 percent of people with PKD will have kidney failure by age 60, and about 60 percent will have kidney failure by age 70. Polycystic kidney disease can also affect the brain or heart causing an aneurysm, but the condition does not occur in 25% of patients. Go to the next page if you knew the correct answer, or click the link images below to further research the concepts in this question (if desired).

Research Concepts: Polycystic Kidney Disease:

We update eBooks quarterly and Apps daily based on user feedback. Please tap flag to report any questions that need improvement.

Question 39: Which of the following is not required during medical research on human subjects?

Choices: 1. Informed consent 2. Compensation 3. Refusal of treatment 4. Choose alternate treatments

Answer: 2 - Compensation Explanations: Human subjects participating in medical research must have informed consent. They can refuse treatment or choose alternate treatments. Some research studies do offer compensation but this is not a legal requirement. Go to the next page if you knew the correct answer, or click the link images below to further research the concepts in this question (if desired).

Research Concepts: Informed Consent:

We update eBooks quarterly and Apps daily based on user feedback. Please tap flag to report any questions that need improvement.

Question 40: Which of the following is not associated with a predisposition to renal cell carcinoma?

Choices: 1. Obesity 2. Female gender 3. Smoking 4. Genetic syndromes

Answer: 2 - Female gender Explanations: Risk factors for renal cell carcinoma include obesity, smoking, chronic dialysis, and male gender, as well as some genetic syndromes. The genetic syndromes include familial renal cell carcinoma, tuberous sclerosis, and von Hippel-Lindau syndrome. Go to the next page if you knew the correct answer, or click the link images below to further research the concepts in this question (if desired).

Research Concepts: Cancer, Renal Cell:

We update eBooks quarterly and Apps daily based on user feedback. Please tap flag to report any questions that need improvement.

Section 2 Question 41: Which is true of the Good Samaritan Laws? Choices: 1. It is the same nationwide 2. It creates an affirmative duty to act 3. It provides limited protection from liability for providing care in certain emergency situations 4. It never applies to licensed medical professionals.

Answer: 3 - It provides limited protection from liability for providing care in certain emergency situations

Explanations: Generally speaking, Good Samaritan laws provide limited immunity for people who provide care in an emergency situation. The laws generally do not apply to an established provider-patient relationship. Most jurisdictions do not create an affirmative duty for a medical provider to act. The laws generally do not provide absolute protection from liability or from being sued. Go to the next page if you knew the correct answer, or click the link images below to further research the concepts in this question (if desired).

Research Concepts: Good Samaritan Laws:

We update eBooks quarterly and Apps daily based on user feedback. Please tap flag to report any questions that need improvement.

Question 42: What is the most common indicator of diabetes insipidus in patients without an underlying diagnosis of physical or mental illness?

Choices: 1. Level of thirst 2. Number of times urinating through the night 3. Color and concentration of urine 4. Number of ounces taken in 8 hours

Answer: 1 - Level of thirst Explanations: Diabetes insipidus should be considered in patients indicating a high level of thirst. Altered thirst mechanisms present in the elderly or those with alteration in mental status. Go to the next page if you knew the correct answer, or click the link images below to further research the concepts in this question (if desired).

Research Concepts: Diabetes Insipidus:

We update eBooks quarterly and Apps daily based on user feedback. Please tap flag to report any questions that need improvement.

Question 43: Which of the following is NOT one of the three most common urinary abnormalities observed in patients with uric acid stone disease?

Choices: 1. Low urinary citrate 2. High urinary uric acid 3. Low urinary pH 4. Low urinary volume

Answer: 1 - Low urinary citrate Explanations: The majority of uric acid stone formers make stones due to high urinary acid loading. Low urinary volume (200 ml 3. Straining to void with no urine output 4. Suprapubic distention with lower abdominal discomfort

Answer: 1 - Urine output of 130 ml on Foley catheter insertion Explanations: PVR >200 ml, straining to void without urine output, and suprapubic distention with lower abdominal discomfort are suggestive of acute urinary retention. 130 ml urine out by Foley catheter does not suggest acute urinary retention. Normal bladder capacity is up to 500 ml. Go to the next page if you knew the correct answer, or click the link images below to further research the concepts in this question (if desired).

Research Concepts: Urinary Retention:

We update eBooks quarterly and Apps daily based on user feedback. Please tap flag to report any questions that need improvement.

Question 180: Which of the following is correct about the development of the urinary system?

Choices: 1. The ventral wall of the coelomic cavity develops urogenital ridges 2. Nephrotomes are formed in the fourth week 3. Nephrotomes grow medially forming nephric tubules 4. The dorsal aorta invades the nephric tubule to form the internal and external glomeruli

Answer: 4 - The dorsal aorta invades the nephric tubule to form the internal and external glomeruli

Explanations: Mesoderm in the cervical region become nephrotomes in the fourth week. Nephrotomes grow laterally and develop a lumen to become nephric tubules. The dorsal aorta invades the nephric tubule to form the internal and external glomeruli. The external glomeruli do not develop, and the mesoderm becomes the nephrogenic tissue cords which in turn become excretory tubules. Go to the next page if you knew the correct answer, or click the link images below to further research the concepts in this question (if desired).

Research Concepts: Embryology, Genitourinary:

We update eBooks quarterly and Apps daily based on user feedback. Please tap flag to report any questions that need improvement.

Question 181: A glomerular filtration rate (GFR) of 45 mL/min in a patient is classified as which of the following stages of kidney disease?

Choices: 1. Stage 1 2. Stage 2 3. Stage 3 4. Stage 4

Answer: 3 - Stage 3 Explanations: Stage 1 kidney disease is indicated by a GFR greater than 90 mL/min. Stage 2 kidney disease is indicated by a GFR between 60-89 mL/min. Stage 3 kidney disease is indicated by a GFR between 30-59 mL/min. Stage 4 kidney disease is indicated by a GFR between 15-29 mL/min. Go to the next page if you knew the correct answer, or click the link images below to further research the concepts in this question (if desired).

Research Concepts: Renal Failure, Chronic:

We update eBooks quarterly and Apps daily based on user feedback. Please tap flag to report any questions that need improvement.

Question 182: Which of the following is not associated with renal tubular acidosis (RTA) type 1 (Distal)?

Choices: 1. Nephrocalcinosis and calcium phosphate renal stones 2. Bone demineralization 3. Sjogren syndrome 4. Hyperkalemia

Answer: 4 - Hyperkalemia Explanations: Renal tubular acidosis (RTA) is associated with hypokalemia, hypocalcemia, and hyperchloremia as well as nephrocalcinosis, deposition of calcium in the substance of the kidney. It is also associated with calcium phosphate urinary stone formation. This is secondary to alkaline urine, hypercalciuria, and very low urinary citrate. Bone demineralization can cause rickets in children and osteomalacia in adults. Sjogren syndrome is also associated with RTA. Go to the next page if you knew the correct answer, or click the link images below to further research the concepts in this question (if desired).

Research Concepts: Renal Tubular Acidosis, Distal:

We update eBooks quarterly and Apps daily based on user feedback. Please tap flag to report any questions that need improvement.

Question 183: A patient presents with Stage III chronic kidney disease due to hypertension. He takes an ACE inhibitor, a thiazide diuretic, and acetaminophen as needed and has a BP of 145/80. Of the following, which is MOST important in preventing the progression of kidney damage for this patient?

Choices: 1. Smoking cessation 2. Discontinuing use of acetaminophen 3. Addition of another antihypertensive 4. Discontinuing the thiazide

Answer: 3 - Addition of another antihypertensive Explanations: The most important action to take is to maximize BP control with a goal of 3 cm) renal abscesses?

Choices: 1. Antibiotics alone for small abscesses, percutaneous or surgical drainage for large renal abscesses 2. Antibiotics and percutaneous or surgical drainage for all renal abscesses if technically possible 3. Antibiotics and percutaneous or surgical drainage for any renal abscess but only if there is failure to clinically improve 4. Percutaneous or surgical drainage for all renal abscesses if technically possible

Answer: 1 - Antibiotics alone for small abscesses, percutaneous or surgical drainage for large renal abscesses

Explanations: Most renal abscesses < 3 cm in stable patients can be treated with antibiotics alone. Duration of antibiotic therapy is not well defined, but generally, parenteral antibiotics are used until patients are clinically stable for 24 to 48 hours, then switched to oral antibiotics for 2 to 4 weeks. Patients with xanthogranulomatous pyelonephritis should be treated with nephrectomy. Acute focal nephritis typically responds to antibiotics alone. Indications to intervene aggressively include persistent infection unresponsive to appropriate antibiotics, impending sepsis, and ongoing hemodynamic instability. The main contraindication to surgical or percutaneous intervention is coagulopathy or uncorrectable anticoagulation. Poor prognostic factors include poorly controlled diabetes mellitus, advanced age, renal anatomic abnormalities, advanced disease, urosepsis. and impaired renal function at presentation. Go to the next page if you knew the correct answer, or click the link images below to further research the concepts in this question (if desired).

Research Concepts: Abscess, Renal Corticomedullary:

We update eBooks quarterly and Apps daily based on user feedback. Please tap flag to report any questions that need improvement.

Question 258: Which of the following is the best treatment for a hemodialysis patient with severe bleeding and type I von Willebrand disease?

Choices: 1. Intranasal DDAVP 2. Fresh frozen plasma 3. Recombinant factor VIII 4. Platelet transfusion

Answer: 1 - Intranasal DDAVP Explanations: Type I von Willebrand disease (vWD) is caused by decreased production of von Willebrand factor. vWF is produced and stored in both platelets and vascular endothelial cells, and DDAVP (or desmopressin) can stimulate the release of available von Willebrand factor in most patients to increase the serum levels and assist in clot formation. A side effects of DDAVP is hyponatremia so serum sodium levels should be monitored in patients receiving frequent DDAVP doses to control bleeding. Fresh frozen plasma contains von Willebrand factor in it and would be effective therapy for severe bleeding however the large volume of infusion would make it a poor choice for a patient on hemodialysis who can get fluid overloaded easily. Cryoprecipitate, however, contains large amounts of von Willebrand factor and can be used for replacement. Endogenous von Willebrand factor in the bloodstream is bound to factor VIII and protects factor III from degradation, and so the best treatment option for bleeding patients with reduced von Willebrand factor or for those who do not respond to DDAVP is replacement with partially purified factor VIII products, which also contain large amounts of von Willebrand factor. Recombinant factor VIII products are composed solely of purified factor VIII and do not contain any von Willebrand factor. Except in cases of type IIB vWD that are associated with thrombocytopenia, platelet counts are generally normal in vWD patients, and so transfusions would not be of any benefit to control bleeding. Go to the next page if you knew the correct answer, or click the link images below to further research the concepts in this question (if desired).

Research Concepts:

Von Willebrand Disease:

We update eBooks quarterly and Apps daily based on user feedback. Please tap flag to report any questions that need improvement.

Question 259: Hemolytic uremic syndrome (HUS) is typically caused by which of the following?

Choices: 1. Enterotoxigenic E. coli 2. Clostridium difficile 3. Enteroaggregative E. coli 4. Enterohemorrhagic E. coli

Answer: 4 - Enterohemorrhagic E. coli Explanations: Enterohemorrhagic E. coli is the cause of hemolytic uremic syndrome (HUS). HUS presents with a triad of microangiopathic hemolytic anemia, thrombocytopenia, and acute renal failure. Classic HUS is caused by Shiga toxin producing E coli. The incidence of HUS is increased during summer and early fall. HUS is common in all races but very rare in blacks. The vast majority of individuals affected are children. HUS typically presents with diarrhea with or without vomiting, irritability, bloody diarrhea, oliguria, and hematuria. Go to the next page if you knew the correct answer, or click the link images below to further research the concepts in this question (if desired).

Research Concepts: Hemolytic Uremic Syndrome (HUS):

We update eBooks quarterly and Apps daily based on user feedback. Please tap flag to report any questions that need improvement.

Question 260: A patient is undergoing work up for an elective cholecystectomy. She is in chronic renal failure, has bilateral peripheral edema, and has symptoms consistent with congestive heart failure. Preoperative labs include a hematocrit of 22%, a potassium level of 6.2 mEq/L, a creatinine of 7.3, a BNP of 850, and chest x-ray shows bilateral pulmonary edema. What is the most appropriate intervention at this time?

Choices: 1. Increase diuresis 2. Placement of an A-V fistula and initiate dialysis 3. Placement of a temporary dialysis catheter and initiate dialysis 4. Stress test

Answer: 3 - Placement of a temporary dialysis catheter and initiate dialysis

Explanations: The patient's clinical picture indicates fluid overload. Medical management will not be successful. An A-V fistula will not mature quickly enough but should have been considered 6 months prior to this presentation. A stress test is not appropriate at this juncture. Go to the next page if you knew the correct answer, or click the link images below to further research the concepts in this question (if desired).

Research Concepts: Renal Failure, Chronic:

We update eBooks quarterly and Apps daily based on user feedback. Please tap flag to report any questions that need improvement.

Question 261: Which of the following would be suspected in a young adult male with chronic sinusitis, hemoptysis, and glomerulonephritis?

Choices: 1. Eosinophilic granulomatosis with polyangiitis 2. Polyarteritis nodosa 3. Polychondritis 4. Granulomatosis with polyangiitis

Answer: 4 - Granulomatosis with polyangiitis Explanations: Granulomatosis with polyangiitis is a very rare autoimmune disorder. The hallmark of the disorder includes necrotizing granulomatosis and inflammation of small and medium sized blood vessels. The classic presentation includes chronic sinusitis, saddle nose deformity, pneumonitis, and pulmonary disease. Renal disease manifests as crescents and necrotizing glomerulonephritis. Go to the next page if you knew the correct answer, or click the link images below to further research the concepts in this question (if desired).

Research Concepts: Granulomatosis with Polyangiitis (GPA, Wegener Granulomatosis):

We update eBooks quarterly and Apps daily based on user feedback. Please tap flag to report any questions that need improvement.

Question 262: A child develops enteritis followed by hemolyticuremic syndrome. What is the most likely cause of the enteritis?

Choices: 1. S. aureus 2. S. typhimurium 3. E. coli 4. V. cholera

Answer: 3 - E. coli Explanations: E. coli has been associated with hemolytic-uremic syndrome. The reaction to the toxin may cause disseminated intravascular coagulation. This, in turn, causes thrombocytopenia, hemolytic anemia, and acute renal failure. It is often associated with consumption of undercooked hamburger meat. Go to the next page if you knew the correct answer, or click the link images below to further research the concepts in this question (if desired).

Research Concepts: Hemolytic Uremic Syndrome (HUS):

We update eBooks quarterly and Apps daily based on user feedback. Please tap flag to report any questions that need improvement.

Question 263: In a patient who has lupus nephritis and is resistant to corticosteroids, what is the next treatment option?

Choices: 1. Cyclophosphamide 2. Methotrexate 3. Cyclosporine 4. Hemodialysis

Answer: 1 - Cyclophosphamide Explanations: The chief goal in lupus nephritis is to normalize renal function or to prevent progression of renal disease. In general, corticosteroids remain the treatment of choice if a patient has clinically significant renal disease. However, if patients are resistant to steroids, one can start cyclophosphamide, azathioprine, or mycophenolate. Hypertension should be treated aggressively with ACE inhibitors and both fat and protein should be restricted in diet. Go to the next page if you knew the correct answer, or click the link images below to further research the concepts in this question (if desired).

Research Concepts: Lupus Nephritis:

We update eBooks quarterly and Apps daily based on user feedback. Please tap flag to report any questions that need improvement.

Question 264: Apart from the kidneys, which of the following organs is frequently involved in autosomal recessive polycystic kidney disease?

Choices: 1. Liver 2. Pancreas 3. Spleen 4. Bladder

Answer: 1 - Liver Explanations: Autosomal recessive polycystic kidney disease occurs in children while the autosomal dominant version occurs in adults. Hepatic cysts may occur in autosomal recessive polycystic kidney disease. Other organ involvement may occur but it is less common. Go to the next page if you knew the correct answer, or click the link images below to further research the concepts in this question (if desired).

Research Concepts: Polycystic Kidney Disease, Childhood:

We update eBooks quarterly and Apps daily based on user feedback. Please tap flag to report any questions that need improvement.

Question 265: You are most likely to see Brewer infarcts in a patient with:

Choices: 1. Urinary tract infections 2. Pyelonephritis 3. Epididymitis 4. Testicular torsion

Answer: 2 - Pyelonephritis Explanations: Brewer infarcts are often seen in patients with pyelonephritis. Go to the next page if you knew the correct answer, or click the link images below to further research the concepts in this question (if desired).

Research Concepts: Pyelonephritis, Acute:

We update eBooks quarterly and Apps daily based on user feedback. Please tap flag to report any questions that need improvement.

Question 266: Select the correct association between the nomenclature and description of the following glomerular disorders:

Choices: 1. Focal segmental glomerulosclerosis - Thick GBM, hypercellular glomeruli 2. Proliferative glomerulonephritis - Only a few glomeruli are abnormal 3. Secondary glomerular disease - Involves only glomeruli and no other target organs 4. Crescentic glomerulonephritis - Proliferation of parietal epithelial cells around glomerulus

Answer: 4 - Crescentic glomerulonephritis - Proliferation of parietal epithelial cells around glomerulus

Explanations: Focal segmental glomerulosclerosis - fibrosis involving only a segment of the involved glomerulus. Proliferative glomerulonephritis - greater than 100 nuclei in affected glomeruli. Secondary glomerular disease - Involves glomeruli and other target organs (i.e. SLE). Crescentic glomerulonephritis is characterized by the proliferation of parietal epithelial cells around the glomerulus and is associated rapid renal failure. Go to the next page if you knew the correct answer, or click the link images below to further research the concepts in this question (if desired).

Research Concepts: Glomerulonephritis, Crescentic:

We update eBooks quarterly and Apps daily based on user feedback. Please tap flag to report any questions that need improvement.

Question 267: Which of the following is not seen on a radiographic examination of a patient with osteomalacia?

Choices: 1. Osteopenia 2. Soft tissue calcification 3. Looser fracture 4. Osteoblastic lesions

Answer: 4 - Osteoblastic lesions Explanations: Radiographic examination of osteomalacia may only reveal osteopenia. One may see coarsened trabeculae, Looser zones, and a rachitic rosary. Other findings include soft tissue calcification, fractures, and osseous resorption. Causes of osteomalacia include vitamin D or phosphorus deficiency, renal tubular acidosis, chronic renal failure, malabsorption, rare tumorinduced osteomalacia, and cadmium poisoning among other causes. Go to the next page if you knew the correct answer, or click the link images below to further research the concepts in this question (if desired).

Research Concepts: Osteomalacia, Renal Osteodystrophy:

We update eBooks quarterly and Apps daily based on user feedback. Please tap flag to report any questions that need improvement.

Question 268: Which of the following is an indication for initiation of hemodialysis in a patient with acute renal failure?

Choices: 1. Creatinine > 7 mg/dL 2. A pericardial friction rub 3. Peripheral pitting edema 4. Hematocrit < 25%

Answer: 2 - A pericardial friction rub Explanations: Indications for hemodialysis include uncontrollable hyperkalemia, pericarditis, severe volume overload, encephalopathy, and severe metabolic acidosis. Uremic pericarditis is an indication for dialysis because it can lead to pericardial tamponade. Go to the next page if you knew the correct answer, or click the link images below to further research the concepts in this question (if desired).

Research Concepts: Renal Failure, Acute:

We update eBooks quarterly and Apps daily based on user feedback. Please tap flag to report any questions that need improvement.

Question 269: On normal-appearing static renal scintigraphic images, why might both renal lower poles appear slightly diminished in activity, gently fading from the corresponding activity in the upper poles?

Choices: 1. Overlying abdominal adipose creates attenuation 2. Bilateral space occupying lesions 3. Adrenal gland attenuation 4. Kidneys are oriented in the body such that the lower poles are slightly anterior to the upper poles, since the imaging is posterior, the lower poles are farther from the detector

Answer: 4 - Kidneys are oriented in the body such that the lower poles are slightly anterior to the upper poles, since the imaging is posterior, the lower poles are farther from the detector

Explanations: Adipose can cause attenuation, but it would be very unusual to be bilaterally symmetric and uniform as described. Space occupying lesions, which are not usually perfectly bilaterally symmetric, typically show sharp cut-offs, not gradual fading of activity as described here. The adrenal glands could not do this. There would have to be large adrenal masses. And the adrenals are superior to, not along the lower poles of, the kidneys. Go to the next page if you knew the correct answer, or click the link images below to further research the concepts in this question (if desired).

Research Concepts: Nuclear Renal Scan:

We update eBooks quarterly and Apps daily based on user feedback. Please tap flag to report any questions that need improvement.

Question 270: A defect in the decarboxylation of the alpha keto acids of the branched-chain amino acids is known as:

Choices: 1. Galactosemia 2. Phenylketonuria 3. Alkaptonuria 4. Maple Syrup Urine Disease

Answer: 4 - Maple Syrup Urine Disease Explanations: The increased branched chain alpha-keto acid excretion in urine is often detected by its odor similar to maple syrup. Go to the next page if you knew the correct answer, or click the link images below to further research the concepts in this question (if desired).

Research Concepts: Maple Syrup Urine Disease:

We update eBooks quarterly and Apps daily based on user feedback. Please tap flag to report any questions that need improvement.

Question 271: A 42-year-old patient with a history chronic sinusitis, hemoptysis, and hematuria has a kidney biopsy showing necrotizing glomerulitis with crescent formation. What is the most likely diagnosis?

Choices: 1. Goodpasture syndrome 2. Granulomatosis with polyangiitis 3. Histoplasmosis 4. Sarcoidosis

Answer: 2 - Granulomatosis with polyangiitis Explanations: Granulomatosis with polyangiitis is secondary to focal sclerosing vasculitis. There is involvement of the sinuses, lung, and kidney. The disease is most common in middle aged men but may occur at any age. 90% of patients are of Northern European descent. Go to the next page if you knew the correct answer, or click the link images below to further research the concepts in this question (if desired).

Research Concepts: Granulomatosis with Polyangiitis (GPA, Wegener Granulomatosis):

We update eBooks quarterly and Apps daily based on user feedback. Please tap flag to report any questions that need improvement.

Question 272: In a patient with autosomal dominant polycystic kidney disease and hypertension, the goal of treatment is to lower blood pressure to what level?

Choices: 1. Less than 160/80 2. 130/88 3. Greater than 100/80 4. 140/90

Photo:Contributed by Scott Dulebohn, MD

Answer: 2 - 130/88 Explanations: The desired blood pressure in patients with autosomal dominant polycystic kidney disease is 130/88. If there is proteinuria, the target blood pressure should be less than 125/75. The lower the blood pressure, the slower the progression of kidney disease. Go to the next page if you knew the correct answer, or click the link images below to further research the concepts in this question (if desired).

Research Concepts: Polycystic Kidney Disease:

We update eBooks quarterly and Apps daily based on user feedback. Please tap flag to report any questions that need improvement.

Question 273: In what condition is the anticentromere antibody found?

Choices: 1. CREST syndrome 2. SLE 3. MCTD 4. Sjogren syndrome

Answer: 1 - CREST syndrome Explanations: Anticentromere antibodies are found in 60-90 percent of patients with CREST syndrome. Anti Scl 90 (anti topoisomerase antibody is also associated with diffuse scleroderma and indicates a poor prognosis. Other findings in CREST patients include elevate calcium, Raynaud phenomenon, esophageal dysmotility and sclerodactyly. A few patients with CREST may have an early rise in ANA levels. Go to the next page if you knew the correct answer, or click the link images below to further research the concepts in this question (if desired).

Research Concepts: Systemic Sclerosis (CREST syndrome):

We update eBooks quarterly and Apps daily based on user feedback. Please tap flag to report any questions that need improvement.

Question 274: Which of the following is not a good initial alternative for anticoagulation of patients who have heparin-induced thrombocytopenia?

Choices: 1. Lepirudin 2. Argatroban 3. Fondaparinux 4. Warfarin

Answer: 4 - Warfarin Explanations: There are a number of recommended alternative anticoagulants to heparin in a patient with heparin-induced thrombocytopenia. Fondaparinux, lepirudin, and argatroban are all useful. Warfarin is not initially recommended for such patients as it can cause micro thromboses. Patients can be transitioned to warfarin after the patient is stably anticoagulated with an alternative agent and platelet count rises above 150,000/microliter. Go to the next page if you knew the correct answer, or click the link images below to further research the concepts in this question (if desired).

Research Concepts: Thrombocytopenia, Heparin Induced:

We update eBooks quarterly and Apps daily based on user feedback. Please tap flag to report any questions that need improvement.

Question 275: How long after placing an indwelling urinary catheter does infection become a major concern?

Choices: 1. 6 hours 2. 24 hours 3. 72 hours 4. 96 hours

Answer: 3 - 72 hours Explanations: Bacterial colonization starts at about 72 hours after urinary catheter placement. Catheter-related urinary tract infection (UTI) occurs because urethral catheters introduce organisms into the bladder and promote colonization by providing a surface for bacterial growth. Irritation of the mucosal area in the urethra further increases the potential for infection. The 2009 Centers for Disease Control and Prevention guidelines for prevention of catheter-associated UTIs recommend catheter use only for appropriate indications. These indications include patients receiving only comfort measures, critical care patients requiring accurate inputs and outputs, operative management, hip fractures, urinary retention, and enlarged prostate with an inability to void. Catheter use and duration should be limited in all patients, especially those at higher risk for catheter-associated UTI (e.g., women, elderly persons, and immunocompromised patients). A catheter should be discontinued as soon as possible following a surgical procedure or stabilization of a critical state. Foley catheters should not be inserted for urinary incontinence management. Around 80 percent of nosocomial infections are related to catheter use. Improved management of catheter-related UTIs was approved as a National Patient Safety Goal in 2012. An order for a Foley catheter should be obtained from the provider prior to insertion. Criteria for its insertion must be reviewed. It is important to insert the catheter using sterile technique and to use a leg guard or securing device to prevent accidental removal or tugging on the catheter. A baseline urinalysis should be sent upon catheter insertion to determine if a UTI already exists. The Foley bag must be dated and initialed. Replacement of Foley catheters is recommended after two weeks if placed for a long-term cause. Patients must be educated on proper care and must not touch the drainage valve to the urinal or graduated cylinder or the toilet when emptying. The Foley drainage bag must always remain below the patient's bladder.

Go to the next page if you knew the correct answer, or click the link images below to further research the concepts in this question (if desired).

Research Concepts: Indwelling Urinary Catheter:

We update eBooks quarterly and Apps daily based on user feedback. Please tap flag to report any questions that need improvement.

Question 276: Where should the catheter ideally be placed for peritoneal dialysis in a female?

Choices: 1. Recto uterine pouch 2. Inside the bladder 3. Right side of the abdomen 4. Anywhere in lower pelvis

Answer: 1 - Recto uterine pouch Explanations: The recto uterine pouch is the ideal place for placement of peritoneal dialysis catheter. The recto uterine pouch is an extension of the peritoneal cavity between the rectum and back wall of the uterus. Go to the next page if you knew the correct answer, or click the link images below to further research the concepts in this question (if desired).

Research Concepts: Dialysis, Peritoneal:

We update eBooks quarterly and Apps daily based on user feedback. Please tap flag to report any questions that need improvement.

Question 277: Idiopathic hypercalciuria is best treated with which of the following?

Choices: 1. Spironolactone 2. Bumetanide 3. Acetazolamide 4. Hydrochlorothiazide

Answer: 4 - Hydrochlorothiazide Explanations: Hydrochlorothiazide increases the reabsorption of calcium in the distal convoluted tubule. It is thus effective to treat idiopathic hypercalciuria to prevent kidney stones. Other treatments for idiopathic hypercalciuria include orthophosphate, bisphosphonates, and sodium cellulose phosphate. Go to the next page if you knew the correct answer, or click the link images below to further research the concepts in this question (if desired).

Research Concepts: Idiopathic Hypercalciuria:

We update eBooks quarterly and Apps daily based on user feedback. Please tap flag to report any questions that need improvement.

Question 278: Which of the following is strongly associated with post-transplant lymphoproliferative disorder?

Choices: 1. Epstein-Barr virus 2. Cytomegalovirus 3. Human papillomavirus 4. HIV

Answer: 1 - Epstein-Barr virus Explanations: Post-transplant lymphoproliferative disorder (PTLD) is strongly associated with Epstein-Barr virus infections. PTLD commonly occurs in Epstein-Barr virus (EBV) antibody negative recipients receiving organs from an EBV-positive donor. In addition to EBV, cytomegalovirus has been implicated in posttransplant lymphoproliferative malignancy. The most common presentation is a lymphoid swelling, usually of the cervical or inguinal lymph nodes. Reduction in immunosuppression and administration of anti-CD 20 monoclonal antibody (rituximab) are standard first-line treatments for PTLD. Go to the next page if you knew the correct answer, or click the link images below to further research the concepts in this question (if desired).

Research Concepts: Transplantation, Posttransplantation Lymphoproliferative Disorders:

We update eBooks quarterly and Apps daily based on user feedback. Please tap flag to report any questions that need improvement.

Question 279: Which of the following viral pathogens has been shown to affect patients with kidney transplants?

Choices: 1. Adenovirus 2. Retrovirus 3. Polyoma 4. RSV

Answer: 3 - Polyoma Explanations: Polyoma virus infections are now being recognized with increased frequency as a cause of graft rejection. Most people have been exposed to this virus by adulthood via the respiratory tract, however, healthy people do not have symptoms. In patients with renal transplant, polyoma virus is increasingly isolated, but often graft function is not compromised. Widespread use of immunosuppressive agents is thought to cause reactivation of this virus. Go to the next page if you knew the correct answer, or click the link images below to further research the concepts in this question (if desired).

Research Concepts: Transplantation, Renal:

We update eBooks quarterly and Apps daily based on user feedback. Please tap flag to report any questions that need improvement.

Question 280: Which of the following abnormalities is NOT seen in Bartter syndrome?

Choices: 1. Hypokalemia 2. Metabolic acidosis 3. Polyuria 4. Polydipsia

Answer: 2 - Metabolic acidosis Explanations: The cause of Bartter syndrome is a primary defect in sodium chloride reabsorption in the medullary thick ascending limb of the loop of Henle. In addition to hypokalemia and metabolic alkalosis, polyuria, polydipsia, and decreased concentrating ability are often present. There is often increased urinary excretion of calcium and mild decrease in magnesium. Go to the next page if you knew the correct answer, or click the link images below to further research the concepts in this question (if desired).

Research Concepts: Bartter Syndrome:

We update eBooks quarterly and Apps daily based on user feedback. Please tap flag to report any questions that need improvement.

Section 8 Question 281: How can one clinically differentiate acute pyelonephritis from pyonephrosis?

Choices: 1. Severity of the flank pain 2. Presence of hematuria 3. Presence of WBC's in the urine 4. It is not possible to differentiate clinically; it requires imaging studies

Answer: 4 - It is not possible to differentiate clinically; it requires imaging studies

Explanations: Pyonephrosis is an infected kidney with its urinary drainage blocked or obstructed by stones or other means. Clinically, the two entities can appear identically and there is no way to determine the difference without imaging. Non-contrast CT scan or ultrasound are necessary to differentiate the two. This is critical as pyonephrosis requires emergency drainage procedures and acute pyelonephritis does not. Go to the next page if you knew the correct answer, or click the link images below to further research the concepts in this question (if desired).

Research Concepts: Pyonephrosis:

We update eBooks quarterly and Apps daily based on user feedback. Please tap flag to report any questions that need improvement.

Question 282: At what age do renal nephrons start do die? Choices: 1. 40 2. 50 3. 60 4. 70

Answer: 1 - 40 Explanations: Normal loss of nephrons starts at age 40. About 20 percent of nephron units are lost by age 80. Go to the next page if you knew the correct answer, or click the link images below to further research the concepts in this question (if desired).

Research Concepts: Renal Dysfunction:

We update eBooks quarterly and Apps daily based on user feedback. Please tap flag to report any questions that need improvement.

Question 283: Which of the following is the best screening test for autosomal dominant polycystic kidney disease?

Choices: 1. Plain abdominal radiograph 2. Ultrasound 3. Intravenous pyelogram 4. Abdominal CT

Answer: 2 - Ultrasound Explanations: Ultrasound is the best screening test to check for autosomal dominant polycystic kidney disease in family members of affected individuals. The test is portable, non-invasive, and inexpensive. Sensitivity is low in early disease and in young patients. Abdominal CT is more accurate but requires ionizing radiation and is more expensive. It is preferred in a more detailed workup of patients with symptoms worrisome for complications of autosomal dominant polycystic kidney disease. Renal MRI eliminates the concern for ionizing radiation and is also an excellent diagnostic modality. Go to the next page if you knew the correct answer, or click the link images below to further research the concepts in this question (if desired).

Research Concepts: Polycystic Kidney Disease:

We update eBooks quarterly and Apps daily based on user feedback. Please tap flag to report any questions that need improvement.

Question 284: Alport syndrome causes renal dysfunction as well as what other complication?

Choices: 1. Anopsia 2. Hearing loss 3. Dizziness 4. Anosmia

Answer: 2 - Hearing loss Explanations: Alport syndrome affects the kidneys and causes hearing loss. Go to the next page if you knew the correct answer, or click the link images below to further research the concepts in this question (if desired).

Research Concepts: Alport Syndrome:

We update eBooks quarterly and Apps daily based on user feedback. Please tap flag to report any questions that need improvement.

Question 285: The diuretic, triamterene, acts on which of the following?

Choices: 1. Androgen receptor 2. Potassium channel 3. Sodium channel 4. Distal tubule

Answer: 3 - Sodium channel Explanations: Triamterene is a potassium sparing diuretic that acts on the sodium channels at the collecting duct. Go to the next page if you knew the correct answer, or click the link images below to further research the concepts in this question (if desired).

Research Concepts: Triamterene:

We update eBooks quarterly and Apps daily based on user feedback. Please tap flag to report any questions that need improvement.

Question 286: A female presents to the ER for the fifth time for a recurrent UTI. She has been on three different antibiotics, but the infections persist. The urine is examined, and the provider feels that there is a problem at the renal proximal tubule. To increase the effect of antibiotics, what is the best thing to do to improve the killing of the organism?

Choices: 1. Alkalinize the urine 2. Acidify the urine 3. Irrigate the ureters 4. Gives antibiotics IV

Answer: 2 - Acidify the urine Explanations: Acidification improves bacterial killing. Go to the next page if you knew the correct answer, or click the link images below to further research the concepts in this question (if desired).

Research Concepts: Recurrent Urinary Tract Infections:

We update eBooks quarterly and Apps daily based on user feedback. Please tap flag to report any questions that need improvement.

Question 287: Which of the following drugs is not commonly associated with a lupus like reaction?

Choices: 1. Phenytoin 2. Isoniazid 3. Procainamide 4. Ciprofloxacin

Answer: 4 - Ciprofloxacin Explanations: Many medications have been implicated in drug-induced lupus, the above medicines plus quinidine and hydralazine are the most commonly implicated; patients often older, kidneys rarely affected. Go to the next page if you knew the correct answer, or click the link images below to further research the concepts in this question (if desired).

Research Concepts: Lupus Erythematosus, Drug-Induced:

We update eBooks quarterly and Apps daily based on user feedback. Please tap flag to report any questions that need improvement.

Question 288: Which is the most common cause of ureteral obstruction following renal transplantation surgery?

Choices: 1. Hematoma 2. Lymphocele 3. Surgical technical error 4. Ureteral ischemia

Answer: 4 - Ureteral ischemia Explanations: Early uretral obstruction is related to ureteral ischemia, too small an intramural antireflux tunnel at the ureteral-vesical attachment site, or direct compression by either a lymphocoele or hematoma. Of these, ureteral ischemia is by far the most common and is responsible for about 90 percent of obstructions. A longer ureteral segment is at greater risk of obstruction as it is more prone to kinking and ischemia. Ureteral obstructions are considered either early (3 months) depending on how long they occur after transplant surgery. The most distal part of the ureter is almost always involved as this is where the ischemia is usually worst being furthest away from the renal artery. Ureteral stenting is often used to try to minimize this effect. Such stents are typically used for the first 4-6 weeks after surgery. Go to the next page if you knew the correct answer, or click the link images below to further research the concepts in this question (if desired).

Research Concepts: Transplantation, Renal:

We update eBooks quarterly and Apps daily based on user feedback. Please tap flag to report any questions that need improvement.

Question 289: A patient with a diagnosis of granulomatosis with polyangiitis should be started on what treatment?

Choices: 1. Splenectomy 2. Prednisone 3. Antibiotics 4. Antibody transfusions

Answer: 2 - Prednisone Explanations: Granulomatosis with polyangiitis is characterized by vasculitis of respiratory tract and kidney. Treatment includes prednisone and cyclophosphamide. Go to the next page if you knew the correct answer, or click the link images below to further research the concepts in this question (if desired).

Research Concepts: Granulomatosis with Polyangiitis (GPA, Wegener Granulomatosis):

We update eBooks quarterly and Apps daily based on user feedback. Please tap flag to report any questions that need improvement.

Question 290: A Grawitz tumor is commonly found in the: Choices: 1. Brain 2. Kidney 3. Lungs 4. Skin

Answer: 2 - Kidney Explanations: Renal cell adenocarcinoma is also known as Grawitz tumor. Go to the next page if you knew the correct answer, or click the link images below to further research the concepts in this question (if desired).

Research Concepts: Cancer, Renal Cell:

We update eBooks quarterly and Apps daily based on user feedback. Please tap flag to report any questions that need improvement.

Question 291: Which of the following diagnostic tests is paired appropriately with its condition?

Choices: 1. Osmotic fragility test-Polycythemia vera 2. Acid hemolysis test-Sickle cell anemia 3. Leukocyte alkaline phosphatase test-Paroxysmal nocturnal hemoglobinuria 4. Sugar water test-Paroxysmal nocturnal hemoglobinuria

Answer: 4 - Sugar water test-Paroxysmal nocturnal hemoglobinuria Explanations: The osmotic fragility test is used to identify RBC membrane defects such as spherocytosis. The acid hemolysis and sugar water tests are for detecting paroxysmal nocturnal hemoglobinuria. The leukocyte alkaline phosphatase test is for detecting CML, polycythemia vera, hairy cell leukemia, aplastic anemia, and Hodgkin's lymphoma. Go to the next page if you knew the correct answer, or click the link images below to further research the concepts in this question (if desired).

Research Concepts: Paroxysmal Nocturnal Hemoglobinuria:

We update eBooks quarterly and Apps daily based on user feedback. Please tap flag to report any questions that need improvement.

Question 292: Which condition often has anticentromere antibodies? Choices: 1. CREST syndrome 2. SLE 3. MCTD 4. Sjogren syndrome

Answer: 1 - CREST syndrome Explanations: Anticentromere antibodies are seen in 50-80 percent of patients with CREST. Anti-topoisomerase antibody is associated with diffuse scleroderma. Other features to make diagnosis include calcinosis, Raynaud, esophageal dysmotility, sclerodactyly, and telangiectasia. Go to the next page if you knew the correct answer, or click the link images below to further research the concepts in this question (if desired).

Research Concepts: Systemic Sclerosis (CREST syndrome):

We update eBooks quarterly and Apps daily based on user feedback. Please tap flag to report any questions that need improvement.

Question 293: What is the most common cause of death associated with acute renal failure?

Choices: 1. Hyponatremia 2. Infection 3. Bleeding 4. Encephalopathy

Answer: 2 - Infection Explanations: Infection is the leading cause of death in acute renal failure. Go to the next page if you knew the correct answer, or click the link images below to further research the concepts in this question (if desired).

Research Concepts: Renal Failure, Acute:

We update eBooks quarterly and Apps daily based on user feedback. Please tap flag to report any questions that need improvement.

Question 294: What is the most likely cancer that occurs in patients with a horseshoe kidney?

Choices: 1. Transitional cancer of the bladder 2. Angiomyolipoma 3. Renal cell cancer 4. Metastatic melanoma

Answer: 3 - Renal cell cancer Explanations: People with a horseshoe kidney are prone to certain cancers. Renal cell cancer is the most common renal malignancy in patients with a horseshoe kidney. However, the overall incidence of renal cell carcinoma is the same as in normal kidneys. Renal transitional cell cancers, Wilms, carcinoid tumors and sarcomas also occur at a higher incidence in horseshoe kidneys than normal kidneys, but the overall incidence is still less than for renal cell cancers. The increased incidence of renal transitional cell cancers in patients with horseshoe kidneys is felt to occur because of chronic irritation from obstruction, renal stones and infection. Go to the next page if you knew the correct answer, or click the link images below to further research the concepts in this question (if desired).

Research Concepts: Horseshoe Kidney:

We update eBooks quarterly and Apps daily based on user feedback. Please tap flag to report any questions that need improvement.

Question 295: Which of the following is the most common sign of renal failure?

Choices: 1. Hypercalcemia 2. Hypocalcemia 3. Hyperkalemia 4. Hypokalemia

Answer: 3 - Hyperkalemia Explanations: Hyperkalemia is common sign of renal failure. Go to the next page if you knew the correct answer, or click the link images below to further research the concepts in this question (if desired).

Research Concepts: Renal Failure:

We update eBooks quarterly and Apps daily based on user feedback. Please tap flag to report any questions that need improvement.

Question 296: In a diabetic who abuses analgesics, what type of renal disorder can occur?

Choices: 1. Pyelonephritis 2. Nephritic syndrome 3. Papillary necrosis 4. Renal cell cancer

Answer: 3 - Papillary necrosis Explanations: Renal papillary necrosis if a form of nephropathy involving the renal papilla. Analgesic nephropathy is a common cause of papillary necrosis. Papillary necrosis also occurs in diabetics and in sickle cell patients. Go to the next page if you knew the correct answer, or click the link images below to further research the concepts in this question (if desired).

Research Concepts: Papillary Necrosis:

We update eBooks quarterly and Apps daily based on user feedback. Please tap flag to report any questions that need improvement.

Question 297: A 75 year old female presents with accelerating chest pain. She has a history of hypertension, diabetes mellitus, hyperlipidemia, renal insufficiency, and atrial fibrillation. Vital signs show temperature 37.8°C, heart rate 98 bpm, blood pressure 190/110, respirations 20, and oxygen saturation 91%. Laboratories show WBC 12.0, hematocrit 30%, BUN 90 mg/dL, and creatine 3.0 mg/dL. Select the contraindication for cardiac catheterization.

Choices: 1. Renal insufficiency 2. Fever 3. Uncontrolled hypertension 4. None of the above

Answer: 4 - None of the above Explanations: There a few if any contraindications to cardiac catheterization except patient refusal. Relative contraindications include ventricular arrhythmias, acute stroke, severe anemia, active GI bleeding, acute renal failure, uncompensated CHF, contrast allergy, and untreated infection. In this patient, hydration and blood pressure control would be appropriate. Go to the next page if you knew the correct answer, or click the link images below to further research the concepts in this question (if desired).

Research Concepts: Angina, Unstable:

We update eBooks quarterly and Apps daily based on user feedback. Please tap flag to report any questions that need improvement.

Question 298: What is the best treatment of disequilibrium syndrome during dialysis?

Choices: 1. Slow the rate of dialysis 2. Elevate the lower extremities 3. Administer an albuterol nebulizer treatment 4. Administer oxygen

Answer: 1 - Slow the rate of dialysis Explanations: Dialysis equilibrium syndrome is a result of sudden changes in osmolality that occur when urea is removed too rapidly, so the rate of dialysis should be slowed. Symptoms include nausea, confusion, and headache. Go to the next page if you knew the correct answer, or click the link images below to further research the concepts in this question (if desired).

Research Concepts: Dialysis, Hemodialysis:

We update eBooks quarterly and Apps daily based on user feedback. Please tap flag to report any questions that need improvement.

Question 299: What causes the toxicity of cyclophosphamide to the kidney and bladder?

Choices: 1. Acrolein 2. Free radicals 3. Kidney stones 4. Direct toxicity

Answer: 1 - Acrolein Explanations: Cyclophosphamide is metabolized by the liver's microsomal enzymes to hydroxy-cyclophosphamide. Target cells convert it to phosphamide mustard that is active and acrolein that is excreted in the urine. Acrolein binds to the bladder mucosa causing inflammation and possible ulceration. About 10 percent of patients develop microscopic hematuria. Approximately 10 percent of people receiving cyclophosphamide experience microscopic hematuria. Patients should be monitored for red tinged urine by dipstick. Cystoscopy may be needed and can show bleeding points, mucosal ulceration, hemorrhage, and/or necrosis. Go to the next page if you knew the correct answer, or click the link images below to further research the concepts in this question (if desired).

Research Concepts: Cyclophosphamide:

We update eBooks quarterly and Apps daily based on user feedback. Please tap flag to report any questions that need improvement.

Question 300: In reading a functional renal scan, activity is noted in the gallbladder. What does this mean?

Choices: 1. There was a misadministration, and the patient was given a hepatobiliary (HIDA) scan 2. The renal agent was Tc99m-DTPA 3. The renal agent was Tc99m-DMSA 4. The renal agent was Tc99m-MAG3

Answer: 4 - The renal agent was Tc99m-MAG3 Explanations: Kidneys are not seen on a hepatobiliary (HIDA) scan unless there was free pertechnetate, and then the thyroid and gastric mucosa would also be seen. Dimercaptosuccinic acid (DMSA) is not a renal functional agent. Diethylenetriaminepentaacetic acid (DTPA) is not taken up by the liver or gallbladder. MAG3 (mercaptoacetyltriglycine) is slightly excreted by the liver and can accumulate in the gallbladder. This is more apparent when renal function is compromised. Go to the next page if you knew the correct answer, or click the link images below to further research the concepts in this question (if desired).

Research Concepts: Technetium 99m Mercaptoacetyltriglycine:

We update eBooks quarterly and Apps daily based on user feedback. Please tap flag to report any questions that need improvement.

Question 301: A 40-year-old woman with chronic headaches is found to have an asymptomatic aneurysm of the middle cerebral artery. It is successfully clipped. Her family history is remarkable in that her mother died suddenly at age 48 and her younger sister had a subarachnoid hemorrhage secondary to an aneurysm. What study should be done on this patient?

Choices: 1. Carotid duplex 2. Echocardiogram 3. Renal ultrasound 4. Holter monitor

Answer: 3 - Renal ultrasound Explanations: Ten percent of patients found to have cerebral aneurysms have a familial predilection. Polycystic kidney disease (PKD) can be associated but < 0.5 percent of patients with subarachnoid hemorrhage have PKD. Aneurysms found in patients with PKD most commonly are at the middle cerebral artery. Go to the next page if you knew the correct answer, or click the link images below to further research the concepts in this question (if desired).

Research Concepts: Polycystic Kidney Disease, Adult:

We update eBooks quarterly and Apps daily based on user feedback. Please tap flag to report any questions that need improvement.

Question 302: A young child is seen with elevated hypertension. He also is found to have large hemangiomas on his face, eyes, and trunk. Work up reveals that he has a pheochromocytoma. What is the greatest risk to the patient?

Choices: 1. Rupture of hemangioma 2. Development of renal cell cancer 3. Stroke 4. Seizures

Answer: 2 - Development of renal cell cancer Explanations: Von Hippel-Lindau causes adenomas of the viscera, especially renal cell hemangioblastoma. There is a defect in VHL tumor suppressor. Go to the next page if you knew the correct answer, or click the link images below to further research the concepts in this question (if desired).

Research Concepts: Von Hippel Lindau Syndrome:

We update eBooks quarterly and Apps daily based on user feedback. Please tap flag to report any questions that need improvement.

Question 303: A patient presents with polyuria. His serum glucose was normal and his urine was found to be dilute. Which of the following lab abnormalities would indicate primary polydipsia?

Choices: 1. Normal serum sodium concentration 2. Normal serum osmolality 3. Low serum sodium concentration 4. High serum sodium concentration

Photo:Contributed by Scott H. Plantz, MD, FAAEM

Answer: 3 - Low serum sodium concentration Explanations: Primary polydipsia is caused by increased water intake. In a polyuric patient, a low plasma sodium concentration at presentation (less than 137 meq/L, due to water overload) is usually indicative of primary polydipsia. A high-normal plasma sodium concentration (greater than 142 meq/L, due to water loss) points toward diabetes insipidus. Primary polydipsia (psychogenic diabetes insipidus, compulsive water drinking) is a condition in which the patient experiences a powerful need to ingest fluid and presents with polydipsia and polyuria and a plasma osmolarity lower than the normal, often lower than 270 mOsm/L. The synthesis, secretion, nephron sensitivity to ADH, and the osmoreceptor response are all functional. Go to the next page if you knew the correct answer, or click the link images below to further research the concepts in this question (if desired).

Research Concepts: Primary Polydipsia:

We update eBooks quarterly and Apps daily based on user feedback. Please tap flag to report any questions that need improvement.

Question 304: Which drug is most likely to interact with lisinopril? Choices: 1. Furosemide 2. Spironolactone 3. Acetazolamide 4. Indapamide

Answer: 2 - Spironolactone Explanations: ACE inhibitors can result in hyperkalemia due to decreased aldosterone production. Aldosterone normally acts to decrease potassium levels. Spironolactone is a potassium sparing diuretic. Administration of lisinopril and spironolactone may lead to hyperkalemia. Go to the next page if you knew the correct answer, or click the link images below to further research the concepts in this question (if desired).

Research Concepts: Spironolactone:

We update eBooks quarterly and Apps daily based on user feedback. Please tap flag to report any questions that need improvement.

Question 305: What drug is frequently used to treat patients with granulomatosis with polyangiitis?

Choices: 1. Cyclosporine 2. Dialysis 3. Captopril 4. Cyclophosphamide

Answer: 4 - Cyclophosphamide Explanations: Cyclophosphamide treats granulomatosis with polyangiitis. Go to the next page if you knew the correct answer, or click the link images below to further research the concepts in this question (if desired).

Research Concepts: Granulomatosis with Polyangiitis (GPA, Wegener Granulomatosis):

We update eBooks quarterly and Apps daily based on user feedback. Please tap flag to report any questions that need improvement.

Question 306: Which of the following is not a reason for using ultrasound in diagnosing polycystic kidney disease?

Choices: 1. Reliable 2. Cost effective 3. Non invasive 4. Tracking progression of the disease

Answer: 4 - Tracking progression of the disease Explanations: The benefits of using ultrasound in diagnosing polycystic kidney disease are that it is reliable, cost-effective, and noninvasive. CT scans and MRI can be used to measure kidney cyst volume and growth in the monitoring of tracking progression of polycystic kidney disease. Go to the next page if you knew the correct answer, or click the link images below to further research the concepts in this question (if desired).

Research Concepts: Polycystic Kidney Disease:

We update eBooks quarterly and Apps daily based on user feedback. Please tap flag to report any questions that need improvement.

Question 307: Which of the following is true of infantile polycystic kidney?

Choices: 1. It is usually unilateral 2. It is not hereditary 3. It is not associated with cystic disease of other organs 4. It may not be diagnosed until early adult life

Answer: 4 - It may not be diagnosed until early adult life Explanations: Infantile polycystic disease (IPCD) is inherited as an autosomal recessive and encompasses two subgroups, one of which is often not diagnosed until late childhood or early adult life. Go to the next page if you knew the correct answer, or click the link images below to further research the concepts in this question (if desired).

Research Concepts: Polycystic Kidney Disease, Childhood:

We update eBooks quarterly and Apps daily based on user feedback. Please tap flag to report any questions that need improvement.

Question 308: Which genetically derived renal condition is the most common cause of renal failure in adults?

Choices: 1. Medullary sponge kidney 2. Autosomal dominant polycystic kidney disease 3. Von Hippel-Lindau disease 4. Tuberous sclerosis

Answer: 2 - Autosomal dominant polycystic kidney disease Explanations: Autosomal dominant polycystic kidney disease (ADPKD) is the most frequent inherited disorder in humans. It accounts for at least 6-7 percent of patients on dialysis in North America. The disorder is characterized by systemic cysts and progression of disease. The disorder is usually seen in the third or fourth decades of life. Go to the next page if you knew the correct answer, or click the link images below to further research the concepts in this question (if desired).

Research Concepts: Polycystic Kidney Disease:

We update eBooks quarterly and Apps daily based on user feedback. Please tap flag to report any questions that need improvement.

Question 309: A patient with acute cystitis will present with which of the following?

Choices: 1. Dysuria 2. Flank pain 3. Chills 4. Vomiting

Answer: 1 - Dysuria Explanations: Acute cystitis often presents with dysuria, urgency, and frequency. Fevers, chills, and vomiting are more indicative of pyelonephritis. Flank pain may indicate ureterolithiasis. The treatment of choice for uncomplicated cystitis is trimethroprimsulfamethazole. Go to the next page if you knew the correct answer, or click the link images below to further research the concepts in this question (if desired).

Research Concepts: Cystitis:

We update eBooks quarterly and Apps daily based on user feedback. Please tap flag to report any questions that need improvement.

Question 310: Secondary Fanconi syndrome is most likely due to: Choices: 1. Tetracycline 2. Amoxicillin 3. Penicillin 4. Sulfonamides

Answer: 1 - Tetracycline Explanations: Fanconi syndrome is characterized by impairment of the proximal tubules of the kidney. This results in certain chemical being excreted instead of reabsorbed. Chemicals that are lost in urine include amino acids, glucose, phosphate, and uric acid. Secondary Fanconi syndrome can result from a variety of drugs including tetracycline and gentamicin. Other drugs that can cause this are cisplatin, 6-mercaptopurine, valproic acid, tenofovir, and adefovir dipivoxil. Go to the next page if you knew the correct answer, or click the link images below to further research the concepts in this question (if desired).

Research Concepts: Fanconi Syndrome:

We update eBooks quarterly and Apps daily based on user feedback. Please tap flag to report any questions that need improvement.

Question 311: Which of the following is the one absolute contraindication to performing peritoneal dialysis?

Choices: 1. Hepatitis B 2. Lobar pneumonia 3. Colostomy 4. Prior cholecystectomy

Answer: 3 - Colostomy Explanations: Peritoneal dialysis performed in the presence of a colostomy can lead to peritonitis. Kidneys are retroperitoneal structures. Go to the next page if you knew the correct answer, or click the link images below to further research the concepts in this question (if desired).

Research Concepts: Dialysis, Peritoneal:

We update eBooks quarterly and Apps daily based on user feedback. Please tap flag to report any questions that need improvement.

Question 312: In granulomatosis with polyangiitis, the disease process involves the upper respiratory tract, the lungs, and what other structure?

Choices: 1. Joints 2. Heart 3. Gastrointestinal tract 4. Kidneys

Answer: 4 - Kidneys Explanations: Granulomatosis with polyangiitis involves localized granulomatous inflammation of the lower or upper respiratory tract mucosa. There may be progression to necrotizing granulomatous vasculitis and glomerulonephritis. It is most common in those of northern European descent and uncommon in African Americans. It is slightly more common in males. It is most commonly diagnosed between the ages of 35 and 55 years but can occur at any age. Go to the next page if you knew the correct answer, or click the link images below to further research the concepts in this question (if desired).

Research Concepts: Granulomatosis with Polyangiitis (GPA, Wegener Granulomatosis):

We update eBooks quarterly and Apps daily based on user feedback. Please tap flag to report any questions that need improvement.

Question 313: Which is not true of nephrocalcinosis? Choices: 1. Secondary hyperparathyroidism is the most common cause 2. It can occur secondary to distal renal tubular acidosis 3. May be due to medullary sponge kidney 4. May be due to hypertension

Answer: 1 - Secondary hyperparathyroidism is the most common cause Explanations: Nephrocalcinosis is a medical disorder where calcium levels in the kidney are increased. This increase in calcium is often discovered incidentally or on a radiological exam. Nephrocalcinosis may present with polyuria, proteinuria, and polydipsia. The most common cause of nephrocalcinosis is primary hyperparathyroidism. Distal renal tubular acidosis is the next most common cause, followed by hypervitaminosis D states. Nephrocalcinosis also occur in renal failure, use of ACEI, high blood pressure and medullary sponge kidney. Go to the next page if you knew the correct answer, or click the link images below to further research the concepts in this question (if desired).

Research Concepts: Nephrocalcinosis:

We update eBooks quarterly and Apps daily based on user feedback. Please tap flag to report any questions that need improvement.

Question 314: A 62-year-old female diabetic presents for colonoscopy but is confused. She is taken to the emergency room where she is noted to have a nonfocal neurologic exam, normal cardiopulmonary exam, but dry mucous membranes. Laboratories show sodium is 123 meq/L and serum osmolality of 272 mmol/kg. Select the most probable cause of her condition.

Choices: 1. Psychogenic polydipsia 2. Hyperosmolar coma 3. Hypovolemia 4. Syndrome of inappropriate secretion of antidiuretic hormone (SIADH)

Answer: 3 - Hypovolemia Explanations: In preparation for colonoscopy, there is often administration of osmotic laxatives. This can result in hypovolemic hyponatremia. Hyperglycemia can lead to hyperosmolar coma, but the patient is hypoosmolar. Syndrome of inappropriate secretion of antidiuretic hormone (SIADH) is unlikely given the signs of dehydration. Go to the next page if you knew the correct answer, or click the link images below to further research the concepts in this question (if desired).

Research Concepts: Hyponatremia:

We update eBooks quarterly and Apps daily based on user feedback. Please tap flag to report any questions that need improvement.

Question 315: For crush injuries resulting in rhabdomyolysis, what is the diuretic of choice?

Choices: 1. Furosemide 2. Thiazide 3. Carbonic anhydrase 4. Mannitol

Answer: 4 - Mannitol Explanations: Mannitol is an osmotic diuretic, which can cause high urine output. Mannitol is occasionally used in conditions of solute overload from severe hemolysis or rhabdomyolysis. Mannitol is also used in the treatment of cerebral edema and acute glaucoma. Go to the next page if you knew the correct answer, or click the link images below to further research the concepts in this question (if desired).

Research Concepts: Rhabdomyolysis:

We update eBooks quarterly and Apps daily based on user feedback. Please tap flag to report any questions that need improvement.

Question 316: A 60-year-old male with atrial fibrillation is involved in a motor vehicle accident sustaining a compound fracture of the right femur. Ten days later, the patient presents with right-sided paralysis, left flank pain, and hematuria. What is the most likely cause of the new findings?

Choices: 1. Fat embolism 2. Emboli from the heart 3. Emboli from leg deep venous thrombosis 4. Cerebrovascular disease and pyelonephritis

Answer: 2 - Emboli from the heart Explanations: Atrial fibrillation can result in systemic emboli causing stroke and renal infarct. Fat emboli from fracture of long bones most often occur within 24 to 48 hours. Fat emboli or thromboembolism from DVT would lodge in the lung, not the systemic circulation. This would only happen if there was a right to left cardiac shunt such as a septal defect. Go to the next page if you knew the correct answer, or click the link images below to further research the concepts in this question (if desired).

Research Concepts: Rhythm, Atrial Fibrillation (A Fib):

We update eBooks quarterly and Apps daily based on user feedback. Please tap flag to report any questions that need improvement.

Question 317: The majority of post renal transplant cancers have been linked to which virus?

Choices: 1. Cytomegalovirus 2. Epstein Barr virus 3. Herpes virus 4. HIV

Answer: 2 - Epstein Barr virus Explanations: Epstein-Barr virus (EBV) is associated with lymphomas especially in patients who have had renal transplants. Hepatitis B and C are linked to hepatocellular carcinoma. Human herpes virus 8 is linked OT Kaposi sarcoma and lymphomas. Human papilloma viruses are linked to cervical, penile, and head an neck cancers. Antiviral medications are often given to renal transplant patients to reduce infections. Go to the next page if you knew the correct answer, or click the link images below to further research the concepts in this question (if desired).

Research Concepts: Transplantation, Renal:

We update eBooks quarterly and Apps daily based on user feedback. Please tap flag to report any questions that need improvement.

Question 318: Laboratories on a patient with Sjogren syndrome show sodium of 140 meq/L, potassium of 3.1 meq/L, chloride of 110 meq/L, and bicarbonate of 15 meq/L. Urine pH is 6.1, sodium 13 meq/L, potassium 11 meq/L, and chloride 11 meq/L. What is the most likely diagnosis?

Choices: 1. Chronic pancreatitis causing malabsorption 2. Type I renal tubular acidosis 3. Type II RTA 4. Type III RTA

Answer: 2 - Type I renal tubular acidosis Explanations: Type I RTA is characterized by urine pH being greater than 5.5, hypokalemia and no bicarbonate in the urine. This is associated with Sjogren syndrome. Complications can include nephrocalcinosis and calcium phosphate stones. Go to the next page if you knew the correct answer, or click the link images below to further research the concepts in this question (if desired).

Research Concepts: Renal Tubular Acidosis:

We update eBooks quarterly and Apps daily based on user feedback. Please tap flag to report any questions that need improvement.

Question 319: Fusion of the lower poles of the kidneys results in the condition known as horseshoe kidney. In this condition:

Choices: 1. The ureters descend in front of the bridge of renal tissue 2. The kidney bridge crosses the aorta between the superior and inferior mesenteric arteries 3. The conjoint kidney remains in the pelvis 4. The renal arteries arise from the common iliac arteries

Answer: 1 - The ureters descend in front of the bridge of renal tissue Explanations: Renal function may be impaired if the ureters are seriously obstructed. Go to the next page if you knew the correct answer, or click the link images below to further research the concepts in this question (if desired).

Research Concepts: Horseshoe Kidney:

We update eBooks quarterly and Apps daily based on user feedback. Please tap flag to report any questions that need improvement.

Question 320: Which of the following is true of the mutation that most frequently cause polycystic kidney disease?

Choices: 1. Most cases follow a predicable pattern 2. In about 10 percent of these cases, an affected person inherits the mutation from one affected parent 3. 90 percent of cases result from new mutations in one of the genes and occur in people with no history of the disorder in their family 4. Autosomal dominant pattern of inheritance

Answer: 4 - Autosomal dominant pattern of inheritance Explanations: Polycystic kidney disease can vary greatly between patients and further mutations can cause rapid growth of cysts and deterioration of the kidneys' function. In about 90 percent of these cases, an affected person inherits the mutation from one affected parent. About 10 percent of cases result from new mutations in one of the genes and occur in people with no history of the disorder in their family. Polycystic kidney disease is an autosomal dominant pattern of inheritance. Go to the next page if you knew the correct answer, or click the link images below to further research the concepts in this question (if desired).

Research Concepts: Polycystic Kidney Disease, Childhood:

We update eBooks quarterly and Apps daily based on user feedback. Please tap flag to report any questions that need improvement.

Section 9 Question 321: Which site is most frequently involved in the initial presentation of patients with granulomatosis with polyangiitis?

Choices: 1. Kidney 2. Lower airway 3. Upper airway 4. Skin

Answer: 3 - Upper airway Explanations: The upper airway is involved at presentation in 73% of patients with granulomatosis with polyangiitis. The lower airway is involved at presentation in 48% of patients with granulomatosis with polyangiitis. Renal, skin, and eye involvement are seen in 20%, 13%, and 15% of granulomatosis with polyangiitis patients at presentation, respectively. Go to the next page if you knew the correct answer, or click the link images below to further research the concepts in this question (if desired).

Research Concepts: Granulomatosis with Polyangiitis (GPA, Wegener Granulomatosis):

We update eBooks quarterly and Apps daily based on user feedback. Please tap flag to report any questions that need improvement.

Question 322: Which of the following is the most common associated finding associated with a horseshoe kidney?

Choices: 1. Renovascular hypertension 2. Ureteropelvic junction obstruction (UPJ) 3. Hydronephrosis 4. Vesicoureteral reflux

Answer: 2 - Ureteropelvic junction obstruction (UPJ) Explanations: Ureteropelvic junction (UPJ) obstruction is associated with horseshoe kidney in up to 35% of patients with the disorder. High insertion of the ureter into the renal pelvis and the distortion from crossing over the isthmus are contributing factors. Obstructive dilation can be differentiated from non-obstructive dilation by use of radioisotope diuretic renal nuclear scans. In children, the most common presenting symptom is a urinary tract infection. Other possible anomalies include hypospadias, undescended testis and bicornate uterus or septate vagina. Go to the next page if you knew the correct answer, or click the link images below to further research the concepts in this question (if desired).

Research Concepts: Horseshoe Kidney:

We update eBooks quarterly and Apps daily based on user feedback. Please tap flag to report any questions that need improvement.

Question 323: Which clinical feature is not a sign of salicylate toxicity?

Choices: 1. Hyperthermia 2. Hyperventilation 3. Metabolic alkalosis 4. Renal failure

Answer: 3 - Metabolic alkalosis Explanations: Salicylate toxicity causes metabolic acidosis. Go to the next page if you knew the correct answer, or click the link images below to further research the concepts in this question (if desired).

Research Concepts: Toxicity, Salicylate (Aspirin):

We update eBooks quarterly and Apps daily based on user feedback. Please tap flag to report any questions that need improvement.

Question 324: A patient with granulomatosis with polyangiitis will have vasculitic involvement of lung and which organ?

Choices: 1. Kidney 2. Liver 3. Pancreas 4. Spleen

Answer: 1 - Kidney Explanations: Granulomatosis with polyangiitis is characterized by vasculitis of respiratory tract and kidney. Diagnostic gold standard is lung biopsy. The sinuses are often involved so septal biopsy can be diagnostic. Go to the next page if you knew the correct answer, or click the link images below to further research the concepts in this question (if desired).

Research Concepts: Granulomatosis with Polyangiitis (GPA, Wegener Granulomatosis):

We update eBooks quarterly and Apps daily based on user feedback. Please tap flag to report any questions that need improvement.

Question 325: The USPSTF recommends a urinalysis for which group of patients?

Choices: 1. Diabetics 2. A person with high blood pressure for 7 years 3. Pregnant females 4. Females with PID

Answer: 3 - Pregnant females Explanations: The USPSTF does not recommend periodic urine screening of any particular group of patients, except those who are pregnant and those who may have symptoms of a urinary tract infection. The American College of Physicians recommends against routine screening of adults for asymptomatic bacteremia with urinalysis or urine culture. Go to the next page if you knew the correct answer, or click the link images below to further research the concepts in this question (if desired).

Research Concepts: Urinalysis:

We update eBooks quarterly and Apps daily based on user feedback. Please tap flag to report any questions that need improvement.

Question 326: Unlike cyclosporine, which condition is not caused by tacrolimus?

Choices: 1. Hyperglycemia 2. Hirsutism 3. Seizures 4. Hyperkalemia

Answer: 2 - Hirsutism Explanations: A significant difference between cyclosporine and tacrolimus is that the latter does not cause hirsutism. They are otherwise similar in side effect profile. They are both nephrotoxic and associated with hyperglycemia. After prolonged use, the drug can induce non-Hodgkin and skin cancers. Tacrolimus can cause a burning sensation during the initial topical application in the eye. The drug does suppress the immune system and can make a patient prone to fungal and viral infections. Go to the next page if you knew the correct answer, or click the link images below to further research the concepts in this question (if desired).

Research Concepts: Tacrolimus:

We update eBooks quarterly and Apps daily based on user feedback. Please tap flag to report any questions that need improvement.

Question 327: A young man presents with hypertension not fully controlled by multiple medications and microscopic hematuria. CT of the abdomen shows a mixed solid/cystic mass in the right kidney and a solid tumor of the left adrenal gland. There are multiple cysts in the left kidney and in the pancreas. Which of the following should be done?

Choices: 1. Scrotal US 2. Brain MRI 3. Ophthalmic exam 4. All of the above

Answer: 2 - Brain MRI Explanations: This patient likely has von Hippel Lindau disease. In addition to renal cell carcinoma, renal cysts, pancreatic cysts, and pheochromocytoma, the patient may suffer from cerebellar hemangioblastoma, retinal hemangioblastoma, and papillary cystadenoma of the epididymis. So all three choices might be beneficial. But the question asked for the MOST beneficial, and learning about a cerebellar tumor is more important than the other options. It can lead to neurological compromise and death. Retinal hemangioblastomas can at worse lead to retinal detachment and visual loss. Go to the next page if you knew the correct answer, or click the link images below to further research the concepts in this question (if desired).

Research Concepts: Von Hippel Lindau Syndrome:

We update eBooks quarterly and Apps daily based on user feedback. Please tap flag to report any questions that need improvement.

Question 328: How is the diagnosis of horseshoe kidney usually made?

Choices: 1. Physical exam 2. Plain x-rays 3. Incidental radiology exam 4. Findings of pain and hypertension

Answer: 3 - Incidental radiology exam Explanations: The majority of cases of horseshoe kidney are discovered incidentally. A few cases are discovered on a physical exam or at surgery. Physical exam may reveal a midline lower abdominal mass. Go to the next page if you knew the correct answer, or click the link images below to further research the concepts in this question (if desired).

Research Concepts: Horseshoe Kidney:

We update eBooks quarterly and Apps daily based on user feedback. Please tap flag to report any questions that need improvement.

Question 329: Patients with tuberous sclerosis are prone to which renal disorder:

Choices: 1. Renal adenocarcinoma 2. Renal artery stenosis 3. Angiomyolipoma 4. Nephrotic syndrome

Answer: 3 - Angiomyolipoma Explanations: Renal manifestations of tuberous sclerosis include angiomyolipomas and renal cysts. Angiomyolipomas are seen in 80% of patients and are composed of smooth muscle, adipose tissue and connective tissue. Renal failure does occur from bleeding angiomyolipomas and is a common cause of death in these patients. Rarely large lesions undergo differentiation to renal cell cancers. Go to the next page if you knew the correct answer, or click the link images below to further research the concepts in this question (if desired).

Research Concepts: Sclerosis, Tuberous:

We update eBooks quarterly and Apps daily based on user feedback. Please tap flag to report any questions that need improvement.

Question 330: Which drug is used for a patient with repeated calcium deposits in the kidney?

Choices: 1. Furosemide 2. Hydrochlorothiazide 3. Mannitol 4. Acetazolamide

Answer: 1 - Furosemide Explanations: Calcium deposits in the kidney can be secondary to hypercalcemia or nephrolithiasis. Treatment for hypercalcemia includes hydration and then furosemide. Then the underlying cause can be addressed. Nephrolithiasis secondary to calcium containing stones is treated with thiazide diuretics. Go to the next page if you knew the correct answer, or click the link images below to further research the concepts in this question (if desired).

Research Concepts: Nephrocalcinosis:

We update eBooks quarterly and Apps daily based on user feedback. Please tap flag to report any questions that need improvement.

Question 331: To prevent urinary tract infections, a patient with an ileal conduit should do which of the following?

Choices: 1. Sterilize the appliance daily 2. Irrigate the stoma daily 3. Drink 2 to 3 liters of fluids per day 4. Cover the stoma with gauze when changing the appliance

Answer: 3 - Drink 2 to 3 liters of fluids per day Explanations: Adequate fluid intake is the most effective action a patient with an ileal conduit can take to prevent stasis, which is a primary cause of urinary tract infection. The appliance is not sterilized daily and the stoma is not irrigated or covered. Go to the next page if you knew the correct answer, or click the link images below to further research the concepts in this question (if desired).

Research Concepts: Bladder, Urinary Diversions And Neobladders:

We update eBooks quarterly and Apps daily based on user feedback. Please tap flag to report any questions that need improvement.

Question 332: Which of the following types of glomerulonephritis is not associated with normal serum complement C4?

Choices: 1. Hemolytic uremic syndrome 2. Membranoproliferative glomerulonephritis type I 3. Post streptococcal glomerulonephritis 4. ANCA associated diseases

Answer: 2 - Membranoproliferative glomerulonephritis type I Explanations: Membranoproliferative glomerulonephritis type I and II may have low C4. Other causes of low C4 glomerulonephritis include lupus nephritis stage IV and endocarditis-associated glomerulonephritis. Go to the next page if you knew the correct answer, or click the link images below to further research the concepts in this question (if desired).

Research Concepts: Glomerulonephritis, Membranoproliferative:

We update eBooks quarterly and Apps daily based on user feedback. Please tap flag to report any questions that need improvement.

Question 333: After prolonged use of angiotensin converting enzyme (ACE) inhibitors, one can develop what type of renal injury?

Choices: 1. Prerenal 2. Intrinsic 3. Postrenal 4. Tubular

Answer: 1 - Prerenal Explanations: ACE inhibitors are associated with prerenal failure. Go to the next page if you knew the correct answer, or click the link images below to further research the concepts in this question (if desired).

Research Concepts: ACE Inhibitors:

We update eBooks quarterly and Apps daily based on user feedback. Please tap flag to report any questions that need improvement.

Question 334: Upon reviewing serial posterior images obtained with 99m-Tc MAG3, one kidney shows persistent renal pelvic activity. Why would the administration of furosemide be the next step?

Choices: 1. It allows testing for possible urinary tract obstruction 2. It allows testing for renovascular causes of hypertension 3. It decreases the overall radiation burden 4. Actually, it is a mistake to give furosemide in this context

Answer: 1 - It allows testing for possible urinary tract obstruction Explanations: This is a classic situation for performing a furosemide renogram for urinary tract obstruction. This has nothing to do with renovascular hypertension workup, which is performed before and after administration of an ACE inhibitor. Giving furosemide (and not voiding) increases bladder radiation. It is not a mistake to give furosemide in this context. Go to the next page if you knew the correct answer, or click the link images below to further research the concepts in this question (if desired).

Research Concepts: Nuclear Renal Scan:

We update eBooks quarterly and Apps daily based on user feedback. Please tap flag to report any questions that need improvement.

Question 335: A 43-year-old female undergoes a renal biopsy and is found to have thyroidization of the kidney. This means the patient has which disorder?

Choices: 1. Chronic pyelonephritis 2. Diabetic nephropathy 3. Metastatic thyroid cancer 4. Ectopic thyroid secretion from kidney

Answer: 1 - Chronic pyelonephritis Explanations: Thyroidization basically means appearance of a thyroid like structure and is indication of chronic pyelonephritis. In chronic pyelonephritis the tubules are dilated with hyaline casts. There may also be patchy areas of chronic inflammatory cells. Go to the next page if you knew the correct answer, or click the link images below to further research the concepts in this question (if desired).

Research Concepts: Pyelonephritis, Chronic:

We update eBooks quarterly and Apps daily based on user feedback. Please tap flag to report any questions that need improvement.

Question 336: Which is not associated with proximal renal tubular acidosis?

Choices: 1. Amyloidosis 2. Vitamin C deficiency 3. Nephrotic syndrome 4. Lithium toxicity

Answer: 4 - Lithium toxicity Explanations: Lithium toxicity is associated with distal renal tubular acidosis. Other processes produce proximal tubular acidosis. Go to the next page if you knew the correct answer, or click the link images below to further research the concepts in this question (if desired).

Research Concepts: Renal Tubular Acidosis, Proximal:

We update eBooks quarterly and Apps daily based on user feedback. Please tap flag to report any questions that need improvement.

Question 337: Why are gouty attacks increased with alcohol consumption?

Choices: 1. Alcohol impacts uric acid excretion 2. Alcohol beverages contain high levels of uric acid 3. Alcohol directly stimulates uric acid precipitation 4. Alcohol prevents breakdown of uric acid

Answer: 1 - Alcohol impacts uric acid excretion Explanations: Uric acid levels increase when alcohol prevents proper excretion or uric acid. Elevated uric acid levels cause crystallization of uric acid, which lodges in the joint spaces. The most common joint to be affected by gout is the first metatarsalphalangeal joint. A low purine diet is beneficial for patients with gout, which includes decreasing consumption of meats, fish, and dried beans. Go to the next page if you knew the correct answer, or click the link images below to further research the concepts in this question (if desired).

Research Concepts: Gout:

We update eBooks quarterly and Apps daily based on user feedback. Please tap flag to report any questions that need improvement.

Question 338: Both triamterene and amiloride are potassium sparing diuretics, but triamterene is more likely to cause what problem not seen with amiloride?

Choices: 1. Increased calcium nephrolithiasis 2. Increased uric acid nephrolithiasis 3. Less diuresis compared to Amiloride 4. Metabolite caused nephrolithiasis

Answer: 4 - Metabolite caused nephrolithiasis Explanations: Triamterene can form urinary stones of triamterene metabolites. This does not happen with amiloride. Such stones would not show up on renal colic (non-contrast) CT scans so IV contrast must be used. In most cases, patients with a propensity to form urinary stones should be on amiloride instead of triamterene. Go to the next page if you knew the correct answer, or click the link images below to further research the concepts in this question (if desired).

Research Concepts: Triamterene:

We update eBooks quarterly and Apps daily based on user feedback. Please tap flag to report any questions that need improvement.

Question 339: Which of the following conditions does not show low serum C3?

Choices: 1. Systemic lupus erythematosus nephritis 2. Focal segmental glomerulonephritis 3. Membranoproliferative glomerulonephritis 4. Acute post Streptococcal glomerulonephritis

Answer: 2 - Focal segmental glomerulonephritis Explanations: Focal segmental glomerulonephritis will show normal C3. Acute but not late post-Streptococcal glomerulonephritis will show low C3. Systemic lupus erythematosus nephritis and membranoproliferative glomerulonephritis will show low C3. Go to the next page if you knew the correct answer, or click the link images below to further research the concepts in this question (if desired).

Research Concepts: Glomerulonephritis:

We update eBooks quarterly and Apps daily based on user feedback. Please tap flag to report any questions that need improvement.

Question 340: Elderly patients may demonstrate spurious elevation of the systolic blood pressure due to vessel sclerosis. Select the clinical feature that is most predictive of this problem:

Choices: 1. A diagonal ear lobe crease 2. A wide pulse pressure 3. A palpable brachial artery when the blood pressure cuff is inflated above the systolic pressure 4. Postural hypotension

Answer: 3 - A palpable brachial artery when the blood pressure cuff is inflated above the systolic pressure

Explanations: Elderly patients may demonstrate spurious hypertension due to sclerotic vessels. A sclerotic brachial artery, which is palpable when the blood pressure cuff is inflated above the systolic pressure, helps identify individuals with this problem. Go to the next page if you knew the correct answer, or click the link images below to further research the concepts in this question (if desired).

Research Concepts: Blood Pressure, Age Related Changes:

We update eBooks quarterly and Apps daily based on user feedback. Please tap flag to report any questions that need improvement.

Question 341: A patient on dialysis for chronic renal disease secondary to bilateral renal artery stenosis. He is anuric. Despite dialysis the patient's potassium rises above 6.1 meq/L several times a week with ECG changes. He reports he is compliant with his medications and diet. Select the appropriate next step.

Choices: 1. Reduce dialysate sodium 2. Reduce dialysate potassium 3. Start bumetanide 4. Arrange for an implanted defibrillator

Answer: 2 - Reduce dialysate potassium Explanations: Standard dialysate has 2.5 meq/L of potassium but this can be reduced based on predialysis potassium. Bumetanide will not have any effect on an anuric patient. A defibrillator is not indicated. Go to the next page if you knew the correct answer, or click the link images below to further research the concepts in this question (if desired).

Research Concepts: Dialysis, Complications:

We update eBooks quarterly and Apps daily based on user feedback. Please tap flag to report any questions that need improvement.

Question 342: A patient presents with general malaise, cough, weight loss, and microscopic hematuria. A chest x-ray reveals multiple cavitary lesions and a renal biopsy reveals a necrotizing glomerulonephritis with crescent formation. The blood work is pending. What is the most likely diagnosis?

Choices: 1. Sarcoidosis 2. Klebsiella pneumonia 3. Tuberculosis 4. Granulomatosis with polyangiitis

Answer: 4 - Granulomatosis with polyangiitis Explanations: Choices 1, 3, and 4 can involve both the lung and the kidney, but the presence of glomerular crescents in the renal histology indicates rapidly progressive glomerulonephritis. This is a severe disease seen with Goodpasture, systemic lupus erythematosus, and granulomatosis with polyangiitis. Untreated, this can lead to acute renal failure and death within months. It is not typically seen with tuberculosis or sarcoid renal disease. Cavitary lesions can be seen with choices 2, 3, and 4. Pulmonary sarcoid is not typically cavitary. Based on the explanations above, the best diagnosis is granulomatosis with polyangiitis. Biopsy can be obtained from the kidney or the nasal sinuses to confirm the diagnosis. A saddle nose is also a feature in 30% of individual with granulomatosis with polyangiitis. The disorder most often occurs in the 3rd or 4th decade of life and is treated with corticosteroids and cyclophosphamide. Go to the next page if you knew the correct answer, or click the link images below to further research the concepts in this question (if desired).

Research Concepts: Granulomatosis with Polyangiitis (GPA, Wegener Granulomatosis):

We update eBooks quarterly and Apps daily based on user feedback. Please tap flag to report any questions that need improvement.

Question 343: The acute kidney injury associated with cholesterol crystal embolism is usually noticed within how many days?

Choices: 1. 2 2. 3-5 3. 10-20 4. 30-40

Answer: 2 - 3-5 Explanations: Cholesterol crystal embolism (CCE) occurs when atheromatous plaques in large arteries release cholesterol emboli and atheromatous debris into the circulation. These become lodged downstream in smaller arterioles (< 200 micrometers in diameter) and cause varying degrees of ischemia of the target organ. The partially occluded arterioles also display an intense giant cell foreign body inflammatory response with fibrosis and eventually complete occlusion. Subsequent waves of arterial embolization are associated with increasing symptoms and signs of the syndrome and may explain that about 20% of cases go unrecognized. CCE is often triggered by invasive angiography, anticoagulant and/or thrombolytic therapy, or aortic trauma (such as Valsalva increase in intra-abdominal pressure when lifting heavy objects). CCE is associated with significant morbidity and mortality. Acute kidney injury (AKI) manifested by increased creatinine is often delayed by 4-6 days after catheterization, as compared to the AKI of contrast nephropathy in which creatinine rises within 24 hours. Go to the next page if you knew the correct answer, or click the link images below to further research the concepts in this question (if desired).

Research Concepts: Cholesterol Emboli:

We update eBooks quarterly and Apps daily based on user feedback. Please tap flag to report any questions that need improvement.

Question 344: Which of the following antacids can be safely used in patients with renal failure?

Choices: 1. Sodium bicarbonate 2. Aluminum hydroxide 3. Magnesium hydroxide 4. Combination aluminum and magnesium hydroxide

Answer: 2 - Aluminum hydroxide Explanations: Patients with renal failure usually have multiple electrolyte imbalances. Antacids containing sodium bicarbonate or magnesium are better avoided in such patients. Antacids containing sodium bicarbonate and magnesium are contraindicated in patients with renal failure. Go to the next page if you knew the correct answer, or click the link images below to further research the concepts in this question (if desired).

Research Concepts: Renal Failure, Chronic:

We update eBooks quarterly and Apps daily based on user feedback. Please tap flag to report any questions that need improvement.

Question 345: What is the most common associated condition in patients with horseshoe kidney?

Choices: 1. Bifid ureters 2. Absent ureter 3. Ureteropelvic junction obstruction 4. Cancer

Answer: 3 - Ureteropelvic junction obstruction Explanations: The most common associated finding in patients with horseshoe kidney is ureteropelvic junction obstruction. The ureteropelvic junction obstruction occurs in 30% of patients and can lead to obstruction of the ureters. Renal stones are also common in patients with horseshoe kidney and are related to the ureteropelvic junction obstruction. Urine stasis also predisposes patients to infection. Go to the next page if you knew the correct answer, or click the link images below to further research the concepts in this question (if desired).

Research Concepts: Horseshoe Kidney:

We update eBooks quarterly and Apps daily based on user feedback. Please tap flag to report any questions that need improvement.

Question 346: A patient presents with hypokalemia, hypophosphatemia and metabolic acidosis. What is the likely diagnosis?

Choices: 1. Rickets 2. Osteomalacia 3. Fanconi syndrome 4. None of the above

Answer: 3 - Fanconi syndrome Explanations: Fanconi syndrome can present with hypokalemia, hypophosphatemia and metabolic acidosis. Rickets in children is a concomitant finding. Osteomalacia in adults is a concomitant finding. Go to the next page if you knew the correct answer, or click the link images below to further research the concepts in this question (if desired).

Research Concepts: Fanconi Syndrome:

We update eBooks quarterly and Apps daily based on user feedback. Please tap flag to report any questions that need improvement.

Question 347: What is the most common type of urinary stone associated with Proteus infection?

Choices: 1. Struvite 2. Calcium 3. Uric acid 4. Cystine

Answer: 1 - Struvite Explanations: Proteus produces urease. It increases urinary pH. This causes struvite stones. Go to the next page if you knew the correct answer, or click the link images below to further research the concepts in this question (if desired).

Research Concepts: Proteus Mirabilis Infections:

We update eBooks quarterly and Apps daily based on user feedback. Please tap flag to report any questions that need improvement.

Question 348: Which renal imaging agent can be imaged in a delayed fashion, after 6 hours?

Choices: 1. Tc99m-MAG3 2. Tc99m-DTPA 3. Tc99m-DMSA 4. Tc99m-GH

Answer: 3 - Tc99m-DMSA Explanations: First note, this is NOT a question of radioactive decay and physical half-life. All choices use the same radionuclide. MAG3, DTPA, and GH assess renal function and are therefore imaged early, allowing collection of data for constructing time-activity curves. DMSA is a structural imaging agent and is best imaged in a delayed fashion to allow background clearance. Go to the next page if you knew the correct answer, or click the link images below to further research the concepts in this question (if desired).

Research Concepts: Nuclear Renal Scan:

We update eBooks quarterly and Apps daily based on user feedback. Please tap flag to report any questions that need improvement.

Question 349: Which of the following substances is excessively excreted in the Fanconi syndrome due to malfunction of renal tubular transport?

Choices: 1. Glucose 2. Amino acids 3. Phosphate 4. All of the above

Answer: 4 - All of the above Explanations: All the named substances are excessively excreted, as well as electrolytes, and the ability to acidify and concentrate urine is also impaired. Go to the next page if you knew the correct answer, or click the link images below to further research the concepts in this question (if desired).

Research Concepts: Fanconi Syndrome:

We update eBooks quarterly and Apps daily based on user feedback. Please tap flag to report any questions that need improvement.

Question 350: Which of the following is the most common age for polycystic kidney disease symptoms to appear?

Choices: 1. Age 10 to 20 2. Age 20 to 30 3. Age 30 to 40 4. Age 40 to 50

Answer: 3 - Age 30 to 40 Explanations: Polycystic kidney disease symptoms most commonly appear from 30 to 40 years of age. Go to the next page if you knew the correct answer, or click the link images below to further research the concepts in this question (if desired).

Research Concepts: Polycystic Kidney Disease, Adult:

We update eBooks quarterly and Apps daily based on user feedback. Please tap flag to report any questions that need improvement.

Question 351: Pyelonephritis is associated with which finding in the urine?

Choices: 1. RBC casts 2. WBC casts 3. Bence Jones protein 4. Waxy casts

Answer: 2 - WBC casts Explanations: Glomerulonephritis is associated with RBC casts. Pyelonephritis is associated with WBC casts. Waxy casts occur with very low urine flow associated with severe, longstanding kidney disease such as renal failure. Due to urine stasis and their formation in dilated ducts, these casts are significantly larger than hyaline casts. They are cylindrical in shape and possess a higher refractive index. Compared to other cast types, they tend to be more rigid, demonstrating sharp edges, fractures, and broken-off ends. Waxy casts are broad casts. a general term used to describe the wider cast produced in a dilated duct as is seen in chronic renal failure. Multiple myeloma is associated with Bence Jones proteins. Go to the next page if you knew the correct answer, or click the link images below to further research the concepts in this question (if desired).

Research Concepts: Pyelonephritis:

We update eBooks quarterly and Apps daily based on user feedback. Please tap flag to report any questions that need improvement.

Question 352: What is nutcracker syndrome? Choices: 1. Compression of the right renal vein between the superior mesenteric artery and the aorta 2. Compression of the left renal vein between the superior mesenteric artery and the aorta 3. Compression of the right renal vein between the inferior mesenteric artery and aorta 4. Compression of the left renal vein between the inferior mesenteric artery and aorta

Answer: 2 - Compression of the left renal vein between the superior mesenteric artery and the aorta

Explanations: Compression of the left renal vein between the superior mesenteric artery and the aorta can lead to hematuria and pain, otherwise known as nutcracker syndrome. There can be elevated left renal vein pressure and collateral vein development. Some patients have intermittent hematuria with or without left flank or abdominal pain. The syndrome occurs in relatively thin patients and adolescents who often have an otherwise healthy medical history. Go to the next page if you knew the correct answer, or click the link images below to further research the concepts in this question (if desired).

Research Concepts: Anatomy, Abdomen, Kidneys, Veins, Renal:

We update eBooks quarterly and Apps daily based on user feedback. Please tap flag to report any questions that need improvement.

Question 353: Which medication can be removed with hemodialysis? Choices: 1. Carbamazepine 2. Phenytoin 3. Valproic acid 4. Phenobarbital

Answer: 4 - Phenobarbital Explanations: Carbamazepine, phenytoin, and valproic acid cannot be removed by dialysis. Phenobarbital is dialyzable. Go to the next page if you knew the correct answer, or click the link images below to further research the concepts in this question (if desired).

Research Concepts: Dialysis, Hemodialysis:

We update eBooks quarterly and Apps daily based on user feedback. Please tap flag to report any questions that need improvement.

Question 354: An older man is admitted to the hospital with respiratory distress and hemoptysis. He had been well until 3 months ago but has lost 10 pounds. The patient's laboratories include complete blood count with white blood cells of 9.8, hemoglobin of 10.5 mg/dL, blood urea nitrogen of 80 mg/dL, and creatinine of 3.0 mg/dL. Urinalysis shows protein to be 2+, 30 red blood cells per high power field, and occasional RBC casts. CT of the chest shows diffuse alveolar infiltrates secondary to hemorrhage. Anti-myeloperoxidase titer is positive at 132 U/mL (normal less than 1.4 U/mL). Which is the most likely diagnosis?

Choices: 1. Granulomatosis with polyangiitis 2. Goodpasture disease 3. Cryoglobulinemia 4. Microscopic polyangiitis

Answer: 4 - Microscopic polyangiitis Explanations: Microscopic polyangiitis (MPA) is associated with antineutrophil cytoplasmic antibodies (ANCAs) of the perinuclear type. There is small vessel vasculitis, and 12 percent of patients present with diffuse alveolar hemorrhage. MPA does not induce granulomatous formation, differentiating it from granulomatosis with polyangiitis. Therapy is with high dose steroids often with the addition of cyclophosphamide that results in a 74 percent 5-year survival. Go to the next page if you knew the correct answer, or click the link images below to further research the concepts in this question (if desired).

Research Concepts: Microscopic Polyangiitis:

We update eBooks quarterly and Apps daily based on user feedback. Please tap flag to report any questions that need improvement.

Question 355: Which is false about renal transplantation? Choices: 1. Donors with malignancy are not candidates 2. ABO compatibility is not necessary for renal transplant 3. Warm ischemic time should not exceed 30-45 minutes 4. Acute rejection typically occurs in the first 3 months

Answer: 2 - ABO compatibility is not necessary for renal transplant Explanations: Donors for kidney should not have a malignancy. Other individuals who are not candidates for donation include elderly, and those with hypertension and diabetes. ABO compatibility is essential to prevent a hyperacute rejection. Warm ischemia time refers to the time when the kidney is obtained from the body and is readied for transplantation. This time should not exceed more than 30-45 minutes. Acute rejection in renal transplant typically occurs in the first 3 months and may present with fever, high WBC and hypertension. Go to the next page if you knew the correct answer, or click the link images below to further research the concepts in this question (if desired).

Research Concepts: Transplantation, Renal:

We update eBooks quarterly and Apps daily based on user feedback. Please tap flag to report any questions that need improvement.

Question 356: Contrast-induced nephropathy will most likely occur in which patient?

Choices: 1. A dehydrated patient with a creatinine of 2.6 2. A patient with chronic glomerulonephritis 3. A patient with Goodpasture syndrome 4. A patient with adult polycystic kidney disease and a creatinine of 1.8

Answer: 1 - A dehydrated patient with a creatinine of 2.6 Explanations: Advanced age, diabetes, repeat exposure to contrast and CHF are other risk factors for contrast induced nephropathy (CIN). CIN is a leading cause of nosocomial acute renal failure. It has a significantly higher risk of in-hospital and 1-year mortality, even among those who don't need dialysis. Most with CIN who die don't succumb to renal failure. A death from preexisting nonrenal complication or procedural complication is more likely. Nonrenal complications of CIN include systemic complications, cardiac complications, and vascular complications. These include ARDS, pulmonary embolism, MI, CABG, hypotension, shock, hematoma, pseudoaneurysm, and stroke. The definition of CIN is renal function impairment within 2-3 days of giving IV contrast. This is quantified as either a 25% increase over baseline in serum creatinine (SCr) or a 0.5 mg/dL increase in absolute value. Go to the next page if you knew the correct answer, or click the link images below to further research the concepts in this question (if desired).

Research Concepts: Contrast Induced Nephropathy:

We update eBooks quarterly and Apps daily based on user feedback. Please tap flag to report any questions that need improvement.

Question 357: Which renal mass is associated with the phakomatosis of von-Hippel Lindau disease?

Choices: 1. Renal cysts 2. Renal cell carcinoma 3. Both 1 and 2 are true 4. Neither 1 nor 2 are true

Answer: 3 - Both 1 and 2 are true Explanations: Renal cysts, typically multiple, are found in 60 percent of von-Hippel Lindau (VHL) patients. Renal cell carcinomas, often multiple, are found in 25-50 percent of VHL patients. Go to the next page if you knew the correct answer, or click the link images below to further research the concepts in this question (if desired).

Research Concepts: Von Hippel Lindau Syndrome:

We update eBooks quarterly and Apps daily based on user feedback. Please tap flag to report any questions that need improvement.

Question 358: A 49 year old female complains of leg cramps, muscle weakness, and headaches. She has no significant medical history. Exam shows mild elevation of blood pressure but is otherwise normal. Laboratories show mild hypernatremia, hypokalemia, normal glucose, BUN, and creatinine. CT show right adrenal gland cortical tumor. Select the most probable hormone abnormalities.

Choices: 1. Elevated fasting cortisol and decreased ACTH 2. Decreased fasting cortisol and increased ACTH 3. Elevated aldosterone and decreased renin 4. Elevated aldosterone and renin

Answer: 3 - Elevated aldosterone and decreased renin Explanations: The patient most likely has primary hyperaldosteronism or Conn syndrome secondary to either an adrenal cortical adenoma or adrenal cortical carcinoma. Another cause of primary hyperaldosteronism is adrenal hyperplasia. All of these conditions will cause decrease renin. Hyperreninemic hyperaldosteronism or secondary hyperaldosteronism can be secondary to a juxtaglomerular tumor, renal artery stenosis, and hyporeabsorption of sodium. Go to the next page if you knew the correct answer, or click the link images below to further research the concepts in this question (if desired).

Research Concepts: Conn Syndrome:

We update eBooks quarterly and Apps daily based on user feedback. Please tap flag to report any questions that need improvement.

Question 359: Which of the following is true about urethra and bladder development?

Choices: 1. The urinary bladder is formed from the urachus 2. An outbudding of the urethra becomes the prostate 3. Ectoderm becomes the epithelium of the urethra 4. Mesoderm of the mesonephric duct becomes the trigone of the bladder

Answer: 2 - An outbudding of the urethra becomes the prostate Explanations: The cloaca divides from weeks four to seven to form the urogenital sinus (UGS). The upper portion becomes the urinary bladder. The pelvic portion of the UGS becomes the prostatic and membranous urethra. Endodermal cells become the epithelium of the urethra. Go to the next page if you knew the correct answer, or click the link images below to further research the concepts in this question (if desired).

Research Concepts: Embryology, Genitourinary:

We update eBooks quarterly and Apps daily based on user feedback. Please tap flag to report any questions that need improvement.

Question 360: Which of the following can diagnose a patient suspected to have granulomatosis with polyangiitis?

Choices: 1. cANCA levels 2. CRP levels 3. Biopsy 4. Urine sedimentation analysis

Answer: 3 - Biopsy Explanations: Granulomatosis with polyangiitis is characterized by vasculitis of respiratory tract and kidney. Diagnostic gold standard is lung biopsy. cANCA alone is not adequate. Go to the next page if you knew the correct answer, or click the link images below to further research the concepts in this question (if desired).

Research Concepts: Granulomatosis with Polyangiitis (GPA, Wegener Granulomatosis):

We update eBooks quarterly and Apps daily based on user feedback. Please tap flag to report any questions that need improvement.

Section 10 Question 361: An 17-year-old female presents confused and uncooperative. Her vital signs are normal except for a respiratory rate of 32. Blood work reveals Na 137 mEq/L, K 2.5 mEq/L, HCO3 13 mEq/L, Cl 119 mEq/L, BUN 16 mg/dL, Creatinine 0.6 mg/dL, Glucose 80 mg/dL, Ca 9.2 mg/dL, urine pH 7.7, and ABG: PO2 94 PCO2 28 pH 7.27. Which of the following is the most likely diagnosis?

Choices: 1. Bowel obstruction 2. Distal renal tubular acidosis 3. Milk alkali syndrome 4. Malabsorption with diarrhea

Answer: 2 - Distal renal tubular acidosis Explanations: The patient has a hyperchloremic non anion gap metabolic acidosis with appropriate respiratory compensation which can be caused by GI bicarbonate losses such as from diarrhea, proximal renal tubular acidosis RTA), hypokalemic distal RTA, type I, and hyperkalemic distal RTA, type IV. The fact that the patient has an elevated urine pH in the setting of acidosis points to a distal RTA because the normal response would be a lowering of urine pH in the setting of metabolic acidosis. Pseudohypoaldosteronism is associated with hyperkalemia and proximal RTA is associated with low urine pH since it is a bicarbonate wasting acidosis. Go to the next page if you knew the correct answer, or click the link images below to further research the concepts in this question (if desired).

Research Concepts: Renal Tubular Acidosis:

We update eBooks quarterly and Apps daily based on user feedback. Please tap flag to report any questions that need improvement.

Question 362: A 16-year-old was healthy until he developed an upper respiratory tract infection. This was followed 3 days later with a high-grade fever, malaise, and a petechial skin rash. In the emergency room he had a blood pressure of 70/40 mmHg, a heart rate of 120 beats per minute, and a white blood cell count of 22. Labs showed sodium 120 mEq/L, potassium 5.8 mEq/L, chloride 89 mEq/L, blood urea nitrogen 34 mg/dL, creatinine 2.0 mg/dL, and glucose 50 mg/dL. Which of the following diagnoses is most likely?

Choices: 1. Acquired immunodeficiency syndrome (AIDS) 2. Toxic shock syndrome 3. Steven Johnson syndrome 4. Waterhouse-Friderichsen syndrome

Answer: 4 - Waterhouse-Friderichsen syndrome Explanations: The patient has Waterhouse-Friderichsen syndrome. The syndrome is caused by a meningococcus, which causes autoimmune destruction of the adrenal glands. Once the adrenal glands are destroyed, the patient will present with hypoglycemia, hyponatremia, and hyperkalemia. Shock and a petechial rash are other features of this syndrome. Go to the next page if you knew the correct answer, or click the link images below to further research the concepts in this question (if desired).

Research Concepts: Waterhouse-Friderichsen Syndrome:

We update eBooks quarterly and Apps daily based on user feedback. Please tap flag to report any questions that need improvement.

Question 363: A patient that has just received a kidney transplant should be taught:

Choices: 1. Signs and symptoms of infection 2. They will need to weigh themselves each week 3. How to measure their abdominal girth 4. How to perform self-catheterization

Answer: 1 - Signs and symptoms of infection Explanations: Transplant teaching should contain information about their medications, signs of infection, rejection, monitoring daily weight, blood pressure, necessary diet and follow up medical instructions. Go to the next page if you knew the correct answer, or click the link images below to further research the concepts in this question (if desired).

Research Concepts: Transplantation, Renal:

We update eBooks quarterly and Apps daily based on user feedback. Please tap flag to report any questions that need improvement.

Question 364: A female undergoes a renal biopsy and immunofluorescence staining reveals a "Wire loop" appearance. The patient may have which disorder?

Choices: 1. Post streptococcal glomerulonephritis 2. Lupus 3. Alport syndrome 4. Minimal change disease

Answer: 2 - Lupus Explanations: A histological hallmark of lupus membranous glomerulonephritis is wire loop lesions. Wire loop lesions signify immune complex deposition along the glomerular basement membrane. Patients with lupus usually present with hematuria or proteinuria as the initial symptoms. Early recognition is vital to prevent end stage renal failure. Go to the next page if you knew the correct answer, or click the link images below to further research the concepts in this question (if desired).

Research Concepts: Systemic Lupus Erythematosus (SLE):

We update eBooks quarterly and Apps daily based on user feedback. Please tap flag to report any questions that need improvement.

Question 365: A 62-year-old female is hospitalized for congestive heart failure and renal failure. An initial exam is remarkable for jugular venous distention; heart sounds show S1S 2 regular with an S3 no murmurs. Lungs have rales ½ way up bilaterally. There is 2+/4 pitting pedal edema. There are waxy papules in the axilla and inguinal region. Admission labs show electrolytes to be normal, but BUN is 85 mg/dL and a creatinine of 6.1 mg/dL. Total protein was 9.2 g/dL, with an albumin of 3.0 g/dL. Hematocrit was 24 percent, and white blood cell and platelet counts were normal. Urinalysis shows 3+ protein but no cells or cellular casts. Echocardiogram shows a thickened left ventricle with preserved systolic function. Select the appropriate diagnostic test.

Choices: 1. Fat pad biopsy 2. Renal ultrasound 3. Right heart catheterization 4. Bone marrow biopsy

Answer: 4 - Bone marrow biopsy Explanations: The waxy papules make the diagnosis likely to be amyloidosis. Serum protein immunoelectrophoresis would be helpful. The high protein makes multiple myeloma a reasonable diagnosis. This would account for the renal failure and echocardiogram findings. Bone marrow biopsy would evaluate for plasma cells and anemia while being 50 to 60 percent sensitive for amyloidosis. Fat pad biopsy is 60 to 80 percent sensitive for amyloidosis. Go to the next page if you knew the correct answer, or click the link images below to further research the concepts in this question (if desired).

Research Concepts: Amyloidosis:

We update eBooks quarterly and Apps daily based on user feedback. Please tap flag to report any questions that need improvement.

Question 366: When compared to baseline exams in a patient with right-sided renal artery stenosis (RAS), what glomerular filtration rate (GFR) will a post-captopril exam show?

Choices: 1. Diminished on the right 2. Increased on the right 3. Increased on the left 4. Unchanged

Answer: 1 - Diminished on the right Explanations: Captopril is an angiotensin-converting enzyme (ACE) inhibitor, and thus a vasodilator. If there is unilateral renal artery stenosis (RAS), that artery does not dilate, but the other artery does. Relative flow to the side of the stenosis decreases, so glomerular filtration rate (GFR) drops. This study does not assess for hypertension. Rather, this study assesses a possible cause of known hypertension. Go to the next page if you knew the correct answer, or click the link images below to further research the concepts in this question (if desired).

Research Concepts: Captopril Renal Scan:

We update eBooks quarterly and Apps daily based on user feedback. Please tap flag to report any questions that need improvement.

Question 367: A male patient has hypertension not controlled on maximum doses of 4 different medications. The diastolic is hardest to control. Hypokalemia has been a problem even with supplementation until diuretics are discontinued. He does not eat licorice. The plasma renin is low. Sodium loading for several days does not lower serum aldosterone. CT of the adrenals shows no masses. The overnight dexamethasone suppression test does not suppress aldosterone. Select the most likely diagnosis.

Choices: 1. Liddle syndrome 2. Conn syndrome 3. Glucocorticoid-remediable aldosteronism 4. Cortical nodular hyperplasia

Answer: 4 - Cortical nodular hyperplasia Explanations: The diagnosis of hyperaldosteronism is suggested by elevated aldosterone in the presence of volume expansion, low renin levels with volume depletion, and the lack of edema even with diastolic elevation. Hyperaldosteronism without a solitary adenoma makes bilateral cortical nodular hyperplasia likely. Conn syndrome is hyperaldosteronism secondary to an adrenal adenoma secreting aldosterone. Liddle syndrome is caused by dysregulation of epithelial sodium channels in the kidney and is also called pseudohyperaldosteronism. Go to the next page if you knew the correct answer, or click the link images below to further research the concepts in this question (if desired).

Research Concepts: Adrenal, Cortical Nodular Hyperplasia:

We update eBooks quarterly and Apps daily based on user feedback. Please tap flag to report any questions that need improvement.

Question 368: A 61-year-old female presents with pedal and periorbital edema progressive over a couple of months. Exam shows normal blood pressure and the above findings. Urinalysis reveals 4+ proteinuria but no other findings. Renal function is normal, but there is decreased albumin. Select the most likely diagnosis.

Choices: 1. Nils disease 2. Membranous glomerulopathy 3. Focal segmental glomerulosclerosis 4. Diabetic nephropathy

Answer: 2 - Membranous glomerulopathy Explanations: The most common cause of nephrotic syndrome in adults is membranous glomerulopathy. It is the cause in 35 percent of those over 60. Diabetic nephropathy can cause proteinuria to nephrotic levels, but the blood pressure would be higher. The patient could have Nils disease, but it is more common in younger patients. Focal segmental glomerulosclerosis is again more common in younger patients. Go to the next page if you knew the correct answer, or click the link images below to further research the concepts in this question (if desired).

Research Concepts: Nephrotic Syndrome:

We update eBooks quarterly and Apps daily based on user feedback. Please tap flag to report any questions that need improvement.

Question 369: A 52-year-old male with mitral valve endocarditis secondary to Streptococcus viridans is discharged to home on intravenous penicillin via central line. Three weeks later he presents with fever. The central line site is clear. His exam is only remarkable for an erythematous maculopapular rash on his legs and trunk. Vital signs are temperature 39.2 degrees C, pulse 100, respirations 16, and blood pressure 140/85 mmHg. Laboratories show a white blood cell count of 13,500/microL with 70 percent polymorphonuclear cells, 14 percent lymphocytes, 4 percent monocytes, and 12 percent eosinophils. The patient's past renal function had been normal, but now his BUN is 60 mg/dL and creatinine 2.3 mg/dL. What is the best test to conduct on this patient?

Choices: 1. Blood and central line tip bacterial cultures 2. Urine for eosinophils 3. Magnetic resonance imaging of the kidneys and renal veins 4. Antistreptolysin O titers

Answer: 2 - Urine for eosinophils Explanations: The patient most likely has allergic interstitial nephritis caused by penicillin. The eosinophilia is the first indicator and further testing with Hansel's stain for eosinophils in the urine is indicated. Antistreptolysin O titers are useful for group A strep causing glomerulonephritis but not S viridans. Renal imaging would show enlarged kidneys, but the renal veins would not have any changes. Go to the next page if you knew the correct answer, or click the link images below to further research the concepts in this question (if desired).

Research Concepts: Nephritis, Interstitial, Allergic:

We update eBooks quarterly and Apps daily based on user feedback. Please tap flag to report any questions that need improvement.

Question 370: A 78-year-old male with a history of congestive heart failure, hyperlipidemia, myelodysplasia, and chronic kidney disease presents with an acute gout attack of his first metatarsal phalangeal joint. His medications include furosemide, lisinopril, aspirin, simvastatin, and atenolol. His last creatinine was 2.6 mg/dL and his uric acid was 8.5 mg/dL. Complete blood count shows white cell count of 5,200, hemoglobin 8.5 g/dL, hematocrit 25.5%, and platelets of 72,000. Select the best treatment for this patient.

Choices: 1. Prednisone 40 mg a day for a week 2. Colchicine 0.6 mg every 6 hours until relief 3. Naproxen sodium 500 mg twice a day 4. Probenecid 500 mg twice a day

Photo:Contributed by Scott Dulebohn, MD

Answer: 1 - Prednisone 40 mg a day for a week Explanations: Oral steroids are the safest treatment for this patient. Colchicine is poorly tolerated in the elderly and is contraindicated given the renal disease and blood dyscrasia. Allopurinol and probenecid are not beneficial for acute gout and can exacerbate the situation. NSAIDS are very effective for acute gout, but, given the patient's renal disease and risk of gastrointestinal bleeding, they should be avoided. Go to the next page if you knew the correct answer, or click the link images below to further research the concepts in this question (if desired).

Research Concepts: Gout:

We update eBooks quarterly and Apps daily based on user feedback. Please tap flag to report any questions that need improvement.

Question 371: Urinary alkalinization helps in the elimination of which of the following drugs?

Choices: 1. Amphetamines 2. Barbiturates 3. Salicylates 4. Both barbiturates AND salicylates

Answer: 4 - Both barbiturates AND salicylates Explanations: Both barbiturates and salicylates have increased eliminations with alkalinization of the urine. Drugs that alkalinize the urine include carbonic anhydrase inhibitors, sodium bicarbonate, and potassium citrate. Urine alkalization reduces renal elimination of amphetamines and other weak bases. Go to the next page if you knew the correct answer, or click the link images below to further research the concepts in this question (if desired).

Research Concepts: Toxicity, Barbiturate:

We update eBooks quarterly and Apps daily based on user feedback. Please tap flag to report any questions that need improvement.

Question 372: Which is not effectively removed by hemodialysis? Choices: 1. Amphetamines 2. Phenobarbital 3. Digoxin 4. Chlorpropamide

Answer: 3 - Digoxin Explanations: Digoxin cannot be removed by dialysis. Amphetamines, phenobarbital, and chlorpropamide are dialyzable. Go to the next page if you knew the correct answer, or click the link images below to further research the concepts in this question (if desired).

Research Concepts: Dialysis, Hemodialysis:

We update eBooks quarterly and Apps daily based on user feedback. Please tap flag to report any questions that need improvement.

Question 373: A 15 year old girl presented with complaints of abdominal pain, cold intolerance, and burning pain in her extremities. Physical examination showed small red papules over trunk and lower limbs. Eye exam revealed tortuosity of vessels and concentric onion skin appearing deposits on retina with no evidence of diabetic retinopathy. Lab tests revealed renal dysfunction with mild proteinuria and hematuria. Renal biopsy was done which revealed enlarged epithelial cells with a foamy appearance and multiple small clear vacuoles. What is the diagnosis?

Choices: 1. Diabetic nephropathy 2. Fabry disease 3. Lupus nephritis 4. Hereditary hemorrhagic telangiectasia

Answer: 2 - Fabry disease Explanations: Renal biopsy would show spectrum of changes in diabetic nephropathy, starting from, mesangial expansion, increased matrix production later leading to, thickening of the glomerular basement membrane, and finally glomerular sclerosis is caused by intraglomerular hypertension. Fabry disease renal biopsy on light microscopy would be enlarged epithelial cells with a foamy appearance and multiple small clear vacuoles from accumulation of glycosphingolipids. Electron microscope findings show vacuoles consist of round lamellated structures called zebra bodies. Lupus nephritis is staged according to the classification revised by the International Society of Nephrology (ISN) and the Renal Pathology Society (RPS) in 2003 are Class I – Minimal mesangial lupus nephritis, Class II – Mesangial proliferative lupus nephritis, Class III – Focal lupus nephritis (active and chronic; proliferative and sclerosing), Class IV – Diffuse lupus nephritis (active and chronic; proliferative and sclerosing; segmental and global), Class V – Membranous lupus nephritis, and Class VI – Advanced sclerosis lupus nephritis. In hereditary hemorrhagic telangiectasia, renal biopsy is not needed. Skin biopsy findings are often helpful in confirming the diagnosis . Punch biopsy is usually adequate. Findings are localized in the dermal upper-horizontal plexus. The classic features are dilated capillaries and new vessel formation. In the dermis, the walls of dilated vessels may be thickened. Go to the next page if you knew the correct answer, or click the link images below to further research the concepts in this question (if desired).

Research Concepts:

Fabry Disease:

We update eBooks quarterly and Apps daily based on user feedback. Please tap flag to report any questions that need improvement.

Question 374: A young man is seen with hypertension. He also is found to have large hemangiomas on his face, eyes, and trunk. Work up reveals that he has a pheochromocytoma. He most likely has which of the following?

Choices: 1. Peutz-Jeghers syndrome 2. Von Hippel-Lindau syndrome 3. Adult polycystic kidney disease 4. Treacher Collins syndrome

Answer: 2 - Von Hippel-Lindau syndrome Explanations: Von Hippel-Lindau shows an autosomal dominant inheritance disorder. It is characterized by hemangioblastomas of the brain and spinal cord, retinal angiomas, angiomas and cysts of the liver and kidney, and renal cell carcinoma, clear cell type. It is also associated with pheochromocytomas and pancreatic neuroendocrine tumors. About 10 percent of people with von Hippel-Lindau syndrome develop endolymphatic sac tumors of the inner ear which can lead to deafness if not treated. Go to the next page if you knew the correct answer, or click the link images below to further research the concepts in this question (if desired).

Research Concepts: Von Hippel Lindau Syndrome:

We update eBooks quarterly and Apps daily based on user feedback. Please tap flag to report any questions that need improvement.

Question 375: Which of the following vasculitis syndromes predominantly affects upper and lower respiratory systems?

Choices: 1. Granulomatosis with polyangiitis 2. Eosinophilic granulomatosis with polyangiitis 3. Kawasaki disease 4. Sjogren syndrome

Answer: 1 - Granulomatosis with polyangiitis Explanations: Granulomatosis with polyangiitis has similar symptoms to that of polyarteritis nodosa. It also commonly affects the upper and lower respiratory systems. Epistaxis or hemoptysis may be a common presentation. Go to the next page if you knew the correct answer, or click the link images below to further research the concepts in this question (if desired).

Research Concepts: Granulomatosis with Polyangiitis (GPA, Wegener Granulomatosis):

We update eBooks quarterly and Apps daily based on user feedback. Please tap flag to report any questions that need improvement.

Question 376: An elderly female presents with complaints of fatigue and swelling, with a history of hypertension, diabetes mellitus type 2, and osteoarthritis. She is on hydrochlorothiazide, metoprolol, glyburide, and ibuprofen. Her physical exam reveals a BP of 155/90, weight of 43 kg, 1+ peripheral edema, no hepatosplenomegaly. The lab results: Na 136 meq/L, K 5.0 meq/L, HCO3 18 meq/L, Cl 108 meq/L, Creatinine 1.6 mg/dL, Glucose 183 mg/dL, and normal liver function panel. What is the most likely cause of her anemia?

Choices: 1. Multi-infarct dementia 2. Chronic kidney disease 3. Lower GI bleed 4. Folate deficiency

Answer: 2 - Chronic kidney disease Explanations: The patient's GFR is 17.5 mL/min when calculated using the Cockraft Gault equation: [(140-age) X Mass (kg)/ (72XCreatinine (mg/dL)] X0.85 if female. Chronic Kidney Disease is defined by GFR < 60 ml/min/1.73 m2 for 3 or months and this patient's GFR of 17.5 places her in Stage 4, severe CKD. This patient's anemia is due to the decreased production of erythropoietin by her damaged kidneys. Go to the next page if you knew the correct answer, or click the link images below to further research the concepts in this question (if desired).

Research Concepts: Anemia, Chronic:

We update eBooks quarterly and Apps daily based on user feedback. Please tap flag to report any questions that need improvement.

Question 377: A 33-year-old male presents to the ER with severe pain radiating from his groin to his pubis. The pain comes in waves and is excruciating. He says he had similar pain before. On examination, he is tachycardic and diaphoretic. Examination reveals significant pain on palpation of the groin and pubic area. Blood work reveals that he has normal levels of electrolytes. Urine reveals that there are some RBCs. He tells you that the doctor told him that he has translucent stones. What is the best way to make a diagnosis of these stones?

Choices: 1. KUB 2. Ultrasound 3. Unenhanced CT of abdomen and pelvis 4. Cystoscopy

Photo:Contributed by Scott Dulebohn, MD

Answer: 3 - Unenhanced CT of abdomen and pelvis Explanations: CT is the most sensitive test to make a diagnosis of urinary tract calculi. It was once taught that KUB detected urinary tract calculi with 85% sensitivity, but recent work using CT as the gold standard shows that is not close to true. Only a minority of calculi are detected on plain film. Even calculi that are radiolucent on plain film can be seen on CT. Only ureteral calculi that form secondary to indinavir therapy for HIV remain "invisible" to CT. CT has replaced IVP for calculus assessment in the urinary tract and most other reasons as well. CT can also assess other renal pathology and can be performed very rapidly. Stone work-ups are done without IV contrast, but most urinary tract CT work ups include IV contrast infusion. Go to the next page if you knew the correct answer, or click the link images below to further research the concepts in this question (if desired).

Research Concepts: Renal Colic, Acute:

We update eBooks quarterly and Apps daily based on user feedback. Please tap flag to report any questions that need improvement.

Question 378: Which of the following would be expected in a diabetic patient with chronic kidney disease?

Choices: 1. Metabolic acidosis 2. Chronic anemia 3. Elevated urine specific gravity 4. Decreased blood urea nitrogen (BUN) and creatinine levels

Answer: 2 - Chronic anemia Explanations: Erythropoietin, produced in the kidneys, stimulates the production of red blood cells. When diabetes causes chronic kidney disease, the erythropoietin levels decline and anemia is common. Metabolic acidosis is a result of chronic kidney disease. An increase in BUN is indicative of chronic kidney disease. Go to the next page if you knew the correct answer, or click the link images below to further research the concepts in this question (if desired).

Research Concepts: Renal Failure, Chronic:

We update eBooks quarterly and Apps daily based on user feedback. Please tap flag to report any questions that need improvement.

Question 379: A female complains of a two-day history of left-hand weakness. 2 months ago, she developed hypertension. She has lost 15 pounds since then due to abdominal pain worse after eating. She was once an intravenous drug user but is now maintained on methadone. Her other medications include hydrochlorothiazide and quinapril. Physical exam is remarkable for wrist drop. Laboratories show erythrocyte sedimentation rate (ESR) of 75 mm/h, an aspartate aminotransferase (AST) of 162 IU/L, and an alanine aminotransferase (ALT) of 182 IU/L. Select the diagnostic test.

Choices: 1. Radial nerve biopsy 2. Hepatitis B surface antigen 3. Anticytoplasmic neutrophil antibodies 4. Mesenteric angiography

Answer: 1 - Radial nerve biopsy Explanations: The most likely diagnosis is polyarteritis nodosa (PAN). While 30 percent of patients with PAN has had hepatitis B, this would not be diagnostic. Mesenteric angiography might show aneurysmal dilation of arteries, but this is not diagnostic. Radial nerve biopsy is the best diagnostic test. Go to the next page if you knew the correct answer, or click the link images below to further research the concepts in this question (if desired).

Research Concepts: Polyarteritis Nodosa:

We update eBooks quarterly and Apps daily based on user feedback. Please tap flag to report any questions that need improvement.

Question 380: What is the most common neurologic complication of chronic renal failure?

Choices: 1. Encephalopathy 2. Peripheral neuropathy 3. Myopathy 4. Cerebellar dysfunction

Answer: 2 - Peripheral neuropathy Explanations: Mixed sensorimotor axonal polyneuropathy is the most common neurologic complication of chronic renal failure. Acute encephalopathy and dementia can develop but are less common. Myopathy is a rare complication. Gait imbalance can result from neuropathy but not usually from cerebellar dysfunction. Go to the next page if you knew the correct answer, or click the link images below to further research the concepts in this question (if desired).

Research Concepts: Neuropathy, Peripheral:

We update eBooks quarterly and Apps daily based on user feedback. Please tap flag to report any questions that need improvement.

Question 381: A patient receiving hemodialysis has a fistula in the forearm. Which of the following is administered to maintain fistula patency?

Choices: 1. Heparin sodium 2. Ampicillin sodium 3. Atropine sulfate 4. Levothyroxine

Answer: 1 - Heparin sodium Explanations: Heparin sodium is an anticoagulant whose impact on fibrin formation helps prevent blood from clotting. Heparin is used during hemodialysis to prevent blood clots from forming, which in turn, prevents peripheral ischemia and strokes. Ampicillin sodium is an anti-infective that kills bacteria by binding to their cell walls. It is not an anticoagulant and therefore does not affect patency of fistulas. Atropine sulfate is an anti-arrhythmic and anticholinergic used to treat bradycardia and decrease secretions pre-operatively. It does not affect patency of fistulas. Levothyroxine (Synthroid) is a synthetic hormone used to manage hypothyroidism by increasing the metabolic rate of body tissues. Levothyroxine does not affect patency of fistulas. Go to the next page if you knew the correct answer, or click the link images below to further research the concepts in this question (if desired).

Research Concepts: Dialysis, Fistula:

We update eBooks quarterly and Apps daily based on user feedback. Please tap flag to report any questions that need improvement.

Question 382: A patient with congestive heart failure develops pain, swelling, and erythema of the left first metatarsophalangeal joint. His medications include furosemide, lisinopril, carvedilol, and aspirin. What are the most likely findings of fluid obtained from arthrocentesis?

Choices: 1. Gram-positive clusters on gram stain 2. White blood cell count greater than 100,000/microliter 3. Strongly negatively birefringent needle-shaped crystals under polarized light microscopy 4. Weakly positively birefringent rhomboidal crystals under polarized light microscopy

Answer: 3 - Strongly negatively birefringent needle-shaped crystals under polarized light microscopy

Explanations: Diuretics can often cause hyperuricemia and gout. The most commonly affected site is the first metatarsophalangeal joint. The pain is severe, monoarticular, erythematous, swollen, and exquisitely tender. WBC count is usually less than 50,000/microliter and if over 100,000 is more likely to represent sepsis. Go to the next page if you knew the correct answer, or click the link images below to further research the concepts in this question (if desired).

Research Concepts: Gout:

We update eBooks quarterly and Apps daily based on user feedback. Please tap flag to report any questions that need improvement.

Question 383: A compliant patient presents for follow up of stage 5 chronic kidney disease. He is without new complaints and exam shows intact mentation, normal cardiac exam, and normal neurologic exam. There is minimal peripheral edema. Laboratories show creatinine of 6.5 mg/dL, potassium 5.0 meq/L, and HCO3 21 meq/L. Which of the following is most appropriate?

Choices: 1. Arrange for peritoneal dialysis 2. Admit the patient for hemodialysis 3. Continue present treatment 4. Urinalysis and culture

Answer: 3 - Continue present treatment Explanations: This patient does not need dialysis. Indications for dialysis are creatine clearance